You are on page 1of 66

OB NOTES

1. How are OB concepts addressed in management functions, roles, and skills? Answer One common thread runs through the functions, roles, and skills of managers: the need to develop people skills if they are going to be effective and successful. Managers get things done through other people. Managers do their work in an organization. Management functions involve managing the organizationplanning and controlling and managing people within the organizationorganizing and leading. Management roles (see Exhibit 1-1) are the parts managers play within an organization and involve their interaction with people. Management skills, as identified by Robert Katz, boil down to three essential management skills: technical, human, and conceptual. These use OB to manage processes and people and to problem solve. 2. Define organizational behavior. Relate it to management. Answer Organizational behavior (abbreviated OB) is a field of study that investigates the impact that individuals, groups, and structure have on behavior within organizations for the purpose of applying such knowledge toward improving an organizations effectiveness. As managers accomplish their work through others, OB provides the tools for guiding the productivity of others, predicting human behavior at work and the perspectives needed to manage individuals from diverse backgrounds. 3. What is an organization? Is the family unit an organization? Explain. Answer An organization is a consciously coordinated social unit, composed of two or more people, which functions on a relatively continuous basis to achieve a common goal or set of goals. The family is a type of organization because it has all the characteristics of an organization. The one variation is that the goals of a family may not be explicit, and therefore students might argue that it is not an organization per se. 4. Identify and contrast the three general management roles. Answer In the late 1960s, Henry Mintzberg discovered three general management roles that had subroles (See Exhibit 1-1). The ten roles can be grouped as being primarily concerned with interpersonal relationships, the transfer of information, and decision making. Interpersonalceremonial and symbolic Figureheadduties are ceremonial and symbolic in nature Leadershiphire, train, motivate, and discipline employees Liaisoncontact outsiders who provide the manager with information. These may be individuals or groups inside or outside the organization. Informationinvolve the collection and dissemination of information Monitorcollect information from organizations and institutions outside their own Disseminatora conduit to transmit information to organizational members Spokespersonrepresent the organization to outsiders Decisionalfocus on making choices Entrepreneurmanagers initiate and oversee new projects that will improve their organizations performance Disturbance handlerstake corrective action in response to unforeseen problems Resource allocatorsare responsible for allocating human, physical, and monetary resources Negotiatordiscuss issues and bargain with other units to gain advantages for their own unit

5. What is a contingency approach to OB? Answer The final model shown in the text, which is a contingency approach, is shown in Exhibit 1-7. The contingency approach refers to situational factors that are variables which moderate the relationship between the independent and dependent variables. There are four key dependent variables (productivity, absenteeism, turnover, and job satisfaction) and a large number of independent variables (for example, motivation, leadership, work processes), organized by level of analysis, that research indicates have varying effects. Because of the large number of independent variables, the study of OB is complex and requires a systematic approach within organizations as we seek to predict the behavior of people at work. 6. Contrast psychology and sociologys contribution to OB. Answer OB is a field of study that investigates the impact that individuals, group, and structure have on behavior within organizations. Both psychology and sociology are concerned with behavior. Psychology is the science of behavior that studies individual behavior and sociology studies people in relation to their fellow human beings. Psychological study in the field of OB has contributed knowledge on a number of topics including: learning, perception, personality, emotions, training, leadership, motivation, job satisfaction, decision making, etc. Sociological study has contributed knowledge on topics such as: group dynamics, teams, organizational culture, organizational theory and structure, communications, power and conflict. 7. Behavior is generally predictable, so there is no need to formally study OB. Why is that statement wrong? Answer Such a casual or commonsense approach to reading others can often lead to erroneous predictions. OB improves managers predictive ability by replacing intuitive o pinions with a more systematic approach. Behavior generally is predictable if we know how the person perceived the situation and what is important to him or her. While peoples behavior may not appear to be rational to an outsider, there is reason to believe it usually is intended to be rational and it is seen as rational by them. There are certain fundamental consistencies underlying the behavior of all individuals that can be identified and then modified to reflect individual differences. These fundamental consistencies allow predictability. When we use the phrase systematic study, we mean looking at relationships, attempting to attribute causes and effects, and basing our conclusions on scientific evidencethat is, on data gathered under controlled conditions and measured and interpreted in a reasonably rigorous manner. 8. What are the three levels of analysis in our OB model? Are they related? If so, how? Answer Individual, group, organization. The three basic levels are analogous to building blocks each level is constructed upon the previous level. Group concepts grow out of the foundation laid in the individual section; we overlay structural constraints on the individual and group in order to arrive at organizational behavior. 9. If job satisfaction is not a behavior, why is it considered an important dependent variable? Answer Job satisfaction is the difference between the amount of rewards workers receive and the amount they believe they should receive. Unlike the other dependent variables, job satisfaction represents an attitude rather than a behavior. It became a primary dependent variable for two reasons: 1) demonstrated relationship to performance factors and 2) the value preferences held by many OB researchers. Managers have believed for years that satisfied employees are more productive. Much evidence questions that assumed causal

relationship. However, it can be argued that advanced societies should be concerned with the quality of life. Ethically, organizations have a responsibility to provide employees with jobs that are challenging and intrinsically rewarding. 10. What are effectiveness and efficiency, and how are they related to organizational behavior? Answer An organization is productive if it achieves its goals (effective) and does so by transferring inputs to outputs at the lowest cost (efficiency). As such, productivity implies a concern for both effectiveness and efficiency. Hospital exampleeffective when it successfully meets the needs of its clientele. It is efficient when it can do so at a low cost. Business firm exampleeffective when it attains its sales or market share goals, but its productivity also depends on achieving those goals efficiently. Achieving productivity through effectiveness and efficiency involves all three levels of an organization, the individual, the group, and the organizational system. OB provides the tools, insights, and ability to predict outcomes needed to balance these two elements. QUESTIONS FOR CRITICAL THINKING 1. Contrast the research comparing effective managers with successful managers. What are the implications from the research for practicing managers? Answer Successful managersNetworking made the largest relative contribution to success. Human resource management activities made the least relative contribution. Effective managers Communication made the largest relative contribution; networking the least. Successful managers do not give the same emphasis to each of those activities as do effective managers. Their emphases are almost the opposite. This finding challenges the historical assumption that promotions are based on performance, vividly illustrating the importance that social and political skills play in getting ahead in organizations. One common thread runs through the functions, roles, skills, and activities approaches to management: managers need to develop their people skills if they are going to be effective and successful. 2. Why do you think the subject of OB might be criticized as being only common sense, when one would rarely hear such a criticism of a course in physics or statistics? Answer Each of us is a student of behavior by nature. Unfortunately, our casual or commonsense approach to reading others can often lead to erroneous predictions. However, we can improve our predictive ability by replacing our intuitive opinions with a more systematic approach. The systematic approach used in this book will uncover important facts and relationships and will provide a base from which more accurate predictions of behavior can be made. Behavior generally is predictable if we know how the person perceived the situation and what is important to him or her. While peoples behavior may not appear to be rational to an outsider, there is reason to believe it usually is intended to be rational and it is seen as rational by them. Systematic study replaces intuition, or those gut feelings about why I do what I do and what makes others tick. 3. Millions of workers have lost their jobs due to downsizing. At the same time, many organizations are complaining that they cannot find qualified people to fill vacancies. How do you explain this apparent contradiction? Answer The nature of the jobs is the key issue. Low-skill jobs are being replaced; high-skill, conceptual,

and technical jobs are increasing. Employees need new or updated skill sets. Organizations will need to develop a strategy for attracting and retaining desired workers which might include targeted recruiting strategies and employee development programs as just two examples of how to meet this crisis. 4. On a 1 to 10 scale measuring the sophistication of a scientific discipline in predicting phenomena, mathematical physics would probably be a 10. Where do you think OB would fall on the scale? Why? Answer Students answers will vary, but the key point is that OB is moving up the scale and is somewhere about a 5. It retains the complexity and unpredictability of being a human science but is using objective and empirical tools in the study of such behaviors to improve the applicability and predictability of its findings. 5. What do you think is the single most critical people problem facing managers today? Give specific support for your position? Answer Students answers will vary. However, that managers must continue to develop his or her communication skills is always of critical importance. Other issues facing managers from an OB perspective are: the changing nature of work (what we do, how we do it do, and where we do it), challenges brought about by technology, finding and retaining skilled workers, motivating workers, stress and organizational change would all rank high on a critical scale confronting managers in todays workplace

1. Which biographical characteristics best predict productivity? Absenteeism? Turnover? Satisfaction? Answer Finding and analyzing the variables that have an impact on employee productivity, absence, turnover, and satisfaction is often complicated. Age The relationship between age and job performance is increasing in importance. Also, the older you get, the less likely you are to quit your job. The age-absence relationship is partially a function of whether the absence is avoidable or unavoidable. In general, older employees have lower rates of avoidable absence. Reviews of the research find that age and job performance are unrelated. Most studies indicate a positive association between age and satisfaction, at least up to age 60. Other studies, however, have found a U-shaped relationship. When the two types, professional and nonprofessional employees, are separated, satisfaction tends to continually increase among professionals as they age, whereas it falls among nonprofessionals during middle age and then rises again in the later years. Gender The evidence suggests that there are few, if any, important differences between men and women that will affect their job performance. There are no consistent male-female differences in problem-solving ability, analytical skills, competitive drive, motivation, sociability, or learning ability. There is a difference in terms of preference for work schedules. Some studies have found that women have higher turnover rates; others have found no difference. There does not appear to be enough information from which to draw meaningful conclusions. The research on absence consistently indicates that women have higher rates of absenteeism than men do. Marital Status There are not enough studies to draw any conclusions about the effect of marital status on productivity. Research consistently indicates that married employees have fewer absences, undergo less turnover, and are more satisfied with their jobs than are their unmarried coworkers. Tenure Studies consistently demonstrate seniority to be negatively related to absenteeism. Tenure is also a potent variable in explaining turnover. Tenure has consistently been found to be negatively related to turnover and has been suggested as one of the single best predictors of turnover. The evidence indicates that tenure and satisfaction are positively related.

2. Assess the validity of using intelligence scores for selecting new employees. Answer IQ tests are designed to ascertain ones general intellectual abilities. The seven most frequently cited dimensions making up intellectual abilities are number aptitude, verbal comprehension, perceptual speed, inductive reasoning, deductive reasoning, spatial visualization, and memory. Jobs differ in the demands they place on incumbents to use their intellectual abilities. The more information-processing demands that exist in a job, the more general intelligence and verbal abilities will be necessary to perform the job successfully. A careful review of the evidence demonstrates that tests that assess verbal, numerical, spatial, and perceptual abilities are valid predictors of job proficiency at all levels of jobs. The major dilemma faced by employers who use mental ability tests is that they may have a negative impact on racial and ethnic groups. 3. Describe the specific steps you would take to ensure that an individual has the appropriate abilities to satisfactorily do a given job. Answer Managers should conduct a careful job assessment so that they can identify key abilities, and then screen applicants for their fit to those job-related needs. The ability-job fit is critically important to employee satisfaction and longevity. 4. Explain classical conditioning. Answer Classical conditioning grew out of experiments conducted at the turn of the century by a Russian physiologist, Ivan Pavlov, to teach dogs to salivate in response to the ringing of a bell. Learning a conditioned response involves building up an association between a conditioned stimulus and an unconditioned stimulus. Classical conditioning is passivesomething happens and we react in a specific way. It is elicited in response to a specific, identifiable event. 5. Contrast classical conditioning, operant conditioning, and social learning. Answer Classical conditioning grew out of experiments conducted at the turn of the century by a Russian physiologist, Ivan Pavlov, to teach dogs to salivate in response to the ringing of a bell. Key concepts are unconditioned stimulus; conditioned stimulus; and conditioned response. Learning a conditioned response involves building up an association between a conditioned stimulus and an unconditioned stimulus. Classical conditioning is passive. Something happens, and we react in a specific way. It is elicited in response to a specific, identifiable event. Operant conditioning argues that behavior is a function of its consequences. People learn to behave to get something they want or to avoid something they do not want. The tendency to repeat such behavior is influenced by reinforcement or lack of reinforcement. Harvard psychologist B.F. Skinners researc h on operant conditioning extensively expanded our knowledge. Any situation in which it is either explicitly stated or implicitly suggested that reinforcements are contingent on some action on your part involves the use of operant learning. Social learning individuals can also learn by observing what happens to other people, by being told about something, as well as by direct experiences. It is an extension of operant conditioning, it also acknowledges

the existence of observational learning and the importance of perception in learning. People respond to how they perceive and define consequences, not to the objective consequences themselves. The influence of models is central. There are four processes: attentional processes, retention processes, motor reproduction processes, and reinforcement processes. 6. How might employees actually learn unethical behavior on their jobs? Answer Probably the biggest issue is that of the role model set by managers and executives. Plus, employees see people all around them engaging in unethical practices. They hear these people, when caught, giving excuses such as everyone does it, or you have to seize every advantage nowadays, or I never thought Id get caught. 7. Describe the four types of intermittent reinforcers. Answer In an intermittent schedule, not every instance of the desirable behavior is reinforced, but reinforcement is given often enough to make the behavior worth repeating. The intermittent payoffs occur just often enough to reinforce behavior. Intermittent techniques be placed into four categories, as shown in Exhibit 2-4. 1) In a fixed-interval reinforcement schedule, rewards are spaced at uniform time intervals. 2) In a variable-interval reinforcement, rewards are distributed in time so that reinforcements are unpredictable. 3) In a fixed-ratio schedule, after a fixed or constant number of responses is given, a reward is initiated. 4) In a variable-ratio schedule, when the reward varies relative to the behavior of the individual, he or she is said to be reinforced. 8. What are the five steps in behavior modification? Answer The typical OB Mod program follows a five-step problem-solving model: Identifying critical behaviors Developing baseline data Identifying behavioral consequences Developing and implementing an intervention strategy Evaluating performance improvement. 9. If you had to take disciplinary action against an employee, how, specifically, would you do it? Answer Every manager will, at some time, have to deal with problem behaviors. Disciplining employees for undesirable behaviors tells them only what not to do. It does not tell them what alternative behaviors are preferred. Appropriate discipline is clear on what is desired as well as having elements of positive reinforcement for doing what is desired. Discipline needs to balance strategies to extinguish undesired behavior with strategies to encourage desired behavior. 10. Describe the four processes in successful social learning. Answer Social learning involves learning by observing what happens to other people, by being told about something, as well as by direct experiences. Four processes determine the influence that a model will have on an individual. Attentional processes. People learn from a model only when they recognize and pay attention to its critical features. Retention processes. A models influence will depend on how well the individual remembers the models action after the model is no longer readily available. Motor reproduction processes. After a person has seen a new behavior by observing the model, the watching must be converted to doing.

Reinforcement processes. Individuals will be motivated to exhibit the modeled behavior if positive incentives or rewards are provided.

QUESTIONS FOR CRITICAL THINKING 1. All organizations would benefit from hiring the smartest people they can get. Do you agree or disagree with this statement? Support your answer. Answer This is clearly the philosophy of Microsoft, and it seems to be working. Most other organizations focus on experience. Microsoft gives extraordinary attention to one single factor intelligence! This has resulted in greater diversity as well. Microsoft believes its greatest asset is the collective intellectual resources of its employees. 2. What do you think is more likely to lead to success on a job a good ability-job fit or personalityorganization fit? Explain. Answer Research seems to indicate the personality-organization fit is critical for managers and executives to be successful. The key to ability-job fit is knowing how people differ in abilities and using that knowledge to increase the likelihood that an employee will perform his or her job well. The answer is not really either/or but both/and. 3. Besides past work history and an employees job performance, what other mitigating factors do you think a manager should use in applying discipline? And doesnt the mere attempt to use mitigating circumstances turn disciplinary action into a political process? Answer This is an important question because of the issues of fairness and the legal requirement of impartiality. On one hand, it is important that discipline is administered fairly and equitably and that discipline is consistent across individuals for similar offenses. At the same time, even our court system recognizes that there are mitigating circumstances that sometimes need to be considered. Managers need to use a consistent processall employees go through the entire process that does include consideration of mitigating circumstances. Having senior managers review any employee discipline case also provides for confirmation or adjusting of the managers original decision. Many large companies require certain levels of review of any employee discipline issue depending on the seriousness of the violation and consequence of the discipline. Finally, as we will see in later chapters, almost anything, almost any decision within an organization can be made a political decision. Uniform processes and training help minimize that eventuality. 4. What abilities do you think are especially important for success in senior-level management positions? Answer Students answers will vary. Some ideas they may have regarding abilities should include: leadership, problem solving, critical thinking skills, excellent communication skills, decision making, etc. They may want to specify level of education and training as well, such as an MBA, etc.

5. What have you learned about learning that could help you to explain the behavior of studen ts in a classroom if (a) the instructor gives only one testa final examination at the end of the course? (b) the instructor gives four exams during the term, all of which are announced on the first day of class? (c) the students grade is based on the results of numerous exams, none of which are announced by the instructor ahead of time? Answer The instructors are using different schedules of reinforcement which may cause the students to react according to the type of reinforcement provided. In (a) the instructor is using fixed-interval, but only once at the end of the course. Because the students are not getting any reinforcement throughout the term, they may not be learning the material as the class progresses. Some students may become anxious and want to know how they are doing, and others may simply wait to the end of the term to study since it doesnt make any difference anyway. In (b) the instructor is using continuous reinforcement at fixed intervals. This provides students with frequent feedback on performance in the class. Students can adjust their inputs (studying, class attendance, etc.) based on the feedback the tests provide. In (c) the instructor is using variable-interval type of reinforcement. Because the course performance measurements are unpredictable, students would need to stay on top of the material to be ready for a test or quiz.

1. Contrast the Veteran, Boomers, Xers, and Nexters classifications with the terminal values identified in the Rokeach Value Survey. Answer The author has integrated a number of recent analyses of work values into a four-stage model. Exhibit 3-3 segments employees by the era in which they entered the workforce. Workers who entered the workforce from the early 1940s through the early 1960s: Their terminal values are a comfortable life and family security. Employees who entered the workforce during the 1960s through the mid-1970s: Their terminal values are freedom and equality. Individuals who entered the workforce from the mid-1970s through the late 1980s: Their terminal values are a sense of accomplishment and social recognition. The Nexters: Their terminal values are true friendship, happiness, and pleasure. An understanding that individuals values differ but tend to reflect the societal values of the period in which they grew up can be a valuable aid in explaining and predicting behavior. 2. Contrast the cognitive and affective components of an attitude. Answer Attitudes are evaluative statements that are either favorable or unfavorable concerning objects, people, or events. A belief is a value statement and the cognitive component of an attitude. It sets the stage for the more critical part of an attitudeits affective component. Affect is the emotional or feeling segment of an attitude. Attitude essentially refers to the affect part of the three components. In contrast to values, attitudes are less stable. In organizations, attitudes are important because they affect job behavior. 3. What is cognitive dissonance, and how is it related to attitudes? Answer Leon Festinger, in the late 1950s, proposed the theory of cognitive dissonance, seeking to explain the linkage between attitudes and behavior. Cognitive dissonance refers to any incompatibility that an individual might perceive between two or more of his/her attitudes, or between his/her behavior and attitudes. Festinger argued that any form of inconsistency is uncomfortable and that individuals will attempt to reduce

the dissonance. The organizational implications are a greater predictability of the propensity to engage in attitude and behavioral change. The greater the dissonanceafter it has been moderated by importance, choice, and rewards factorsthe greater the pressures to reduce it. 4. What is self-perception theory? How does it increase our ability to predict behavior? Answer Self-perception theory argues that attitudes are used to make sense out of an action. Contrary to cognitive dissonance theory, attitudes are just casual verbal statements. Researchers achieve higher correlations by pursuing whether or not behavior influences attitudes. When asked about an attitude toward some object, individuals recall their behavior relevant to that object and then infer their attitude from their past behavior. While the traditional attitude-behavior relationship is generally positive, the behaviorattitude relationship is stronger. 5. What contingency factors can improve the statistical relationship between attitudes and behavior? Answer Job satisfaction is an individuals general attitude toward his/her job. Jobs require interaction with co-workers and bosses, following organizational rules and policies, meeting performance standards, living with working conditions that are often less than ideal, and the like. This means that an employees assessment of how satisfied or dissatisfied he or she is with his/her job is a complex summation of a number of discrete job elements. Recent research has demonstrated that attitudes significantly predict future behavior. The most powerful moderators are: The importance of the attitude reflects fundamental values, self-interest, or identification with individuals or groups that a person values. Its specificity. The more specific the attitude and the more specific the behavior, the stronger the link between the two. Its accessibility. Attitudes that are easily remembered are more likely to predict behavior than attitudes that are not accessible in memory. Whether there exist social pressures. Discrepancies between attitudes and behavior are more likely to occur where social pressures to behave in certain ways hold exceptional power. Whether a person has direct experience with the attitude. The attitude -behavior relationship is likely to be much stronger if an attitude refers to an individuals direct personal experience. 6. What explains the recent declines in employee job satisfaction? Answer There has been a decline in job satisfaction since the early 1990snearly an eight percent drop in the US alone. This happened despite that fact that those years were ones of growth and economic expansion. Factors that might explain the decline despite growth: Increased productivity through heavier employee workloads and tighter deadlines and the general feeling by employees that they have less control over their work. While some segments of the market are more satisfied than others, they tend to be higher paid, have higher skilled jobs, which gives workers more control and challenges. 7. Are happy workers productive workers? Answer Not necessarily. While happy workers are not necessarily productive workers, the evidence suggests that productivity is likely to lead to satisfaction. Organizations with more satisfied employees tend to be more effective than organizations with less satisfied employees. It might be true that happy organizations are more productive.

8. What is the relationship between job satisfaction and absenteeism? Turnover? Which is the stronger relationship? Answer Managers interest in job satisfaction tends to center on its effect on employee performance. Research shows a consistent negative relationship between satisfaction and absenteeism. It makes sense that dissatisfied employees are more likely to miss work, but other factors have an impact on the relationship and reduce the correlation coefficient. Satisfaction is also negatively related to turnover, but the correlation is stronger than what we found for absenteeism. Again, other factors such as labor market conditions, expectations about alternative job opportunities, and length of tenure with the organization are important constraints on the actual decision to leave ones current job. Evidence indicates that an important moderator of the satisfaction-turnover relationship is the employees level of performance. 9. How can managers get employees to more readily accept working with colleagues who are different from themselves? Answer Managers can initiate diversity programs that include a self-evaluation phase. People are pressed to examine themselves and to confront ethnic and cultural stereotypes they might hold. Additional activities designed to change attitudes include arranging for people to do volunteer work in community or social service centers in order to meet face to face with individuals and groups from diverse backgrounds and using exercises that let participants feel what it is like to be different. 10. Contrast exit, voice, loyalty, and neglect as employee responses to job dissatisfaction. Answer Exhibit 3-6 reviews these four responses and how they differ from one another along two dimensions: constructiveness/destructiveness and activity/passivity. Exit: Behavior directed toward leaving the organization, including looking for a new position as well as resigning. Voice: Actively and constructively attempting to improve conditions, including suggesting improvements, discussing problems with superiors, and some forms of union activity. Loyalty: Passively but optimistically waiting for conditions to improve, including speaking up for the organization in the face of external criticism and trusting the organization and its management to do the right thing. Neglect: Passively allowing conditions to worsen, including chronic absenteeism or lateness, reduced effort, and increased error rate. Exit and neglect behaviors encompass our performance variablesproductivity, absenteeism, and turnover. Voice and loyalty constructive behaviors allow individuals to tolerate unpleasant situations or to revive satisfactory working conditions. QUESTIONS FOR CRITICAL THINKING 1. Thirty-five years ago, young employees we hired were ambitious, conscientious, hardworking, and honest. Todays young workers do not have the same values. Do you agree or disagree with this managers comments? Support your position. Answer This question represents the differences in perception and opinion among cohorts. Including nontraditional students or guests in this discussion will create an enlightening experience for both sides. Students should note the different influences on each of the cohorts and how those influences are demonstrated in their behavior. An understanding that individuals values differ but tend to reflect the societal values of the period in which they grew up can be a valuable aid in explaining and predicting behavior. Employees in their late 30s and

60s, for instance, are more likely to be conservative and accepting of authority than their existential coworkers in their early 50s. Workers under 35 are more likely than the other groups to balk at having to work overtime or weekends and more prone to leave a job in mid-career to pursue another that provides more leisure time. See Exhibit 3-3. 2. Do you think there might be any positive and significant relationship between the possession of certain personal values and successful career progression in organizations like Merrill Lynch, the AFL-CIO, and the city of Clevelands police department? Discuss. Answer The position of your text is yes, you would, for several reasons. People tend to gravitate toward jobs that are compatible with their interests, values, and abilities. Values lay the foundation for the understanding of attitudes and motivation and because they influence our perceptions. Values generally influence attitudes and behavior. 3. Managers should do everything they can to enhance the job satisfaction of their employees. Do you agree or disagree? Support your position. Answer Students will probably argue for yes, however, the reality is there may not be any productivity or bottom line positive reason for doing so. In the real world, managers interest in job satisfaction tends to center on its effect on employee performance. Happy workers are not necessarily productive workers, but evidence suggests that productivity is likely to lead to satisfaction. Also, organizations with more satisfied employees tend to be more effective than organizations with less satisfied employees. It might be true that happy organizations are more productive. Also, it makes sense that dissatisfied employees are more likely to miss work, but other factors have an impact on the relationship and reduce the correlation coefficient. Reference the Sears, Roebuck study in the text. 4. Discuss the advantages and disadvantages of using regular attitude surveys to monitor employee job satisfaction. Answer Using attitude surveys on a regular basis provides managers with valuable feedback on how employees perceive their working conditions. Policies and practices that management views as objective and fair may be seen as inequitable by employees in general or by certain groups of employees. Employee behaviors are often based on perceptions, not reality. The use of regular attitude surveys can alert management to potential problems and employees intentions early so that action can be ta ken to prevent repercussions. On the other hand, the use of surveys may create an expectation for change that may or may not actually come about. 5. When employees are asked whether they would again choose the same work or whether they would want their children to follow in their footsteps, typically less than half answer in the affirmative. What, if anything, do you think this implies about employee job satisfaction? Answer This could indicate something relevant, such as attitude survey results are not accurate; it could reflect something totally irrelevant to what is being measured, i.e., parents desire for the children to do better than they are, so they wish for a better professional career for their children. Or it could reflect that, while the

parents like their jobs, they know that their careers are not good matches for them. This question is too broad to draw accurate conclusions. 1. What is personality? Answer A dynamic concept describing the growth and development of a persons whole psychological system. Personality looks at some aggregate whole that is greater than the sum of the parts. Personality is the dynamic organization within the individual of those psychophysical systems that determine his unique adjustments to his environment. Personality is the sum total of ways in which an individual reacts to and interacts with others. It is most often described in terms of measurable traits that a person exhibits. 2. What behavioral predictions might you make if you knew that an employee had (a) an external locus of control? (b) a low Mach score? (c) low self-esteem? (d) a Type A personality? Answer a) Individuals who rate high in externality are less satisfied with their jobs, have higher absenteeism rates, are more alienated from the work setting, and are less involved on their jobs than are internals. Externals are more compliant and willing to follow directions. Externals should do well on jobs that are well structured and routine and in which success depends heavily on complying with the direction of others. b) An individual high in Machiavellianism is pragmatic, maintains emotional distance, and believes that ends can justify means. High Machs manipulate more, win more, are persuaded less, and persuade others more. High Mach outcomes are moderated by situational factors. High Machs flourish when they interact face to face with others rather than indirectly, when the situation has a minimum number of rules and regulations, thus allowing latitude for improvisation, and when emotional involvement with details irrelevant to winning distracts low Machs. High Machs make good employees in jobs that require bargaining skills or that offer substantial rewards for winning. c) Self-esteemthe degree to which they like or dislike themselves. It is directly related to expectations for success. Individuals with high self-esteem will take more risks in job selection and are more likely to choose unconventional jobs than people with low self-esteem. The most generalizable finding is that low SEs are more susceptible to external influence than are high SEs. Low SEs are dependent on the receipt of positive evaluations from others. In managerial positions, low SEs will tend to be concerned with pleasing others. d) A Type A personality is aggressively inv olved in a chronic, incessant struggle to achieve more and more in less and less time, and, if required to do so, against the opposing efforts of other things or other persons. They: Are always moving, walking, and eating rapidly; Feel impatient with the rate at which most events take place; Strive to think or do two or more things at once; Cannot cope with leisure time; Are obsessed with numbers, measuring their success in terms of how many or how much of everything they acquire. 3. What is the Myers-Briggs Type Indicator? Answer The Myers-Briggs Type Indicator is one of the most widely used personality frameworks MyersBriggs Type Indicator (MBTI). It is a 100-question personality test that asks people how they usually feel or act in particular situations. Individuals are classified as Extroverted or introverted (E or I). Sensing or intuitive (S or N). Thinking or feeling (T or F).

Perceiving or judging (P or J). These classifications are then combined into sixteen personality types. There is no hard evidence that the MBTI is a valid measure of personality. 4. Describe the factors in the Big Five model. Which factor shows the greatest value in predicting behavior? Why does it? Answer Research supports that five basic dimensions underlie all other personality dimensions. The five basic dimensions are: Extraversion: Comfort level with relationships. Extraverts tend to be gregarious, assertive, and sociable. Introverts tend to be reserved, timid, and quiet. Agreeableness: Individuals propensity to defer to others. Highly agreeable peoplecooperative, warm, and trusting. Low agreeableness peoplecold, disagreeable, and antagonistic. Conscientiousness: A measure of reliability. A high conscientious person is responsible, organized, dependable, and persistent. Those who score low on this dimension are easily distracted, disorganized, and unreliable. Emotional stability: A persons ability to withstand stress. People with positive emotional stability tend to be calm, self-confident, and secure. Those with high negative scores tend to be nervous, anxious, depressed, and insecure. Openness to experience: The range of interests and fascination with novelty. Extremely open people are creative, curious, and artistically sensitive. Those at the other end of the openness category are conventional and find comfort in the familiar. Of the five factors conscientiousness predicted job performance across all occupational groups. Individuals who are dependable, reliable, careful, thorough, able to plan, organized, hardworking, etc. tend to have a high job performance record. 5. Do people from the same country have a common personality type? Explain. Answer The Big Five model translates across almost all cross-cultural studies. There are no common personality types for a given country. There is evidence that cultures differ in terms of peoples relationship to their environment. In North America, people believe that they can dominate their environment. People in Middle Eastern countries believe that life is essentially preordained. The prevalence of Type A personalities will be somewhat influenced by the culture in which a person grows up. There are more in capitalistic countries. In cultures such as Sweden and France, where materialism is less revered, we would predict a smaller proportion of Type A personalities. 6. Why might managers today pay more attention to the person-organization fit than the person-job fit? Answer It is most important for an organization facing a dynamic and changing environment, and requiring employees who are able to readily change tasks and move fluidly between teams. It argues that people leave jobs that are not compatible with their personalities. Matching people to the organizational culture at the time of hiring should result in higher employee satisfaction and reduced turnover. 7. What is emotional labor, and why is it important to understanding OB? Answer Emotional labor is when an employee expresses organizationally desired emotions during interpersonal transactions. Originally developed in relation to service jobs, but now seems to apply to every job. For example, you are expected to be courteous and not hostile in interactions with coworkers.

8. How does national culture influence expressed emotions? Answer Cultural norms in the United States dictate that employees in service organizations should smile and act friendly when interacting with customers, but this norm does not apply worldwide. Cultures differ in terms of the interpretation they give to emotions. There tends to be high agreement on what emotions mean within cultures, but not between. Studies indicate that some cultures lack words for such standard emotions as anxiety, depression, or guilt. 9. What is emotional intelligence, and why is it important? Answer Emotional intelligence (EI) refers to an assortment of non-cognitive skills, capabilities, and competencies that influence a persons ability to succeed in coping with environmental demands and pressures. Self-awareness: Being aware of what you are feeling. Self-management: The ability to manage ones own emotions and impulses. Self-motivation: The ability to persist in the face of setbacks and failures. Empathy: The ability to sense how others are feeling. Social skills: The ability to handle the emotions of others. Several studies suggest EI may play an important role in job performance. QUESTIONS FOR CRITICAL THINKING 1. Heredity determines personality. (a) Build an argument to support this statement. (b) Bu ild an argument against this statement. Answer Personality is a dynamic concept describing the growth and development of a persons whole psychological system. Personality is the sum total of ways in which an individual reacts to and interacts with others. It is most often described in terms of measurable traits that a person exhibits. The early argument was that personality was the result of heredity or of environment. It now appears that personality appears to be a result of both influences and even a third factorthe situation. Heredity refers to those factors that were determined at conception. The heredity approach argues that the ultimate explanation of an individuals personality is the molecular structure of the genes, located in the chromosomes. Three different streams of research lend some credibility to the heredity argument. The genetic underpinnings of human behavior and temperament among young children The study of twins who were separated at birth The consistency in job satisfaction over time and across situations Personality characteristics are not completely dictated by heredity. Factors that exert pressures on our personality formation are: The culture in which we are raised Early conditioning Norms among our family Friends, and social groups The environment we are exposed to plays a substantial role in shaping our personalities. Culture establishes the norms, attitudes, and values passed from one generation to the next and creates consistencies over time. Heredity sets the parameters or outer limits, but an individuals full potential will be determined by how well he or she adjusts to the demands and requirements of the environment.

2. The type of job an employee does moderates the relationship between personality and jo b productivity. Do you agree or disagree with this statement? Discuss. Answer Students can take either position but their discussion should include elements of the data of person-organizational fit. Hollands research is central to this discussion. He p resents six personality types and proposes that satisfaction and the propensity to leave a job depend on the degree to which individuals successfully match their personalities to an occupational environment. Each one of the six personality types has a congruent occupational environment. The theory argues that satisfaction is highest and turnover lowest when personality and occupation are in agreement. There does appear to be intrinsic differences in personality among individuals. There are different types of jobs. People in job environments congruent with their personality types should be more satisfied and less likely to voluntarily resign than should people in incongruent jobs. 3. One day, your boss comes in, and hes nervous, edgy, and argumentative. Th e next day, he is calm and relaxed. Does this behavior suggest that personality traits are not consistent from day to day? Answer No, these are simply variations in his emotional state. Emotions are reactions to an object, not a trait. They are object-specific. Emotions have a number of dimensions: variety, intensity, frequency and duration. 4. What, if anything, can managers do to manage emotions? Answer They cannot control the emotions of their colleagues and employees. Emotions are a natural part of an individuals makeup. Where managers err is if they ignore the emotional elements in organizational behavior and assess individual behavior as if it were completely rational. Managers who understand the role of emotions will significantly improve their ability to explain and predict individual behavior. 5. Give some examples of situations where the overt expression of emotions might enhance job performance. Answer Emotions can also enhance performance in two ways. First, emotions can increase arousal levels, thus acting as motivators to higher performance. Second, emotional labor recognizes that feelings can be part of a jobs required behavior. So, for instance, the ability to effectively manage emotions in leadership and sales positions may be critical to success in those positions. 1. Define perception. Answer Perception is a process by which individuals organize and interpret their sensory impressions in order to give meaning to their environment. What one perceives can be substantially different from objective reality. Perception is important in the study of OB because peoples behavior is based on their perception of what reality is, not on reality itself. 2. What is attribution theory? What are its implications for explaining organizational behavior? Answer Attribution theory suggests that when we observe an individuals behavior, we attempt to determine whether it was internally or externally caused. That determination depends largely on three factors: distinctiveness, consensus, and consistency.

3. How are our perceptions of our own actions different from our perceptions of the actions of others? Answer One of the more interesting findings from attribution theory is that there are errors or biases that distort attributions. There is substantial evidence that we have a tendency to underestimate the influence of external factors and overestimate the influence of internal or personal factors. This is the fundamental attribution error. There is also a tendency for individuals to attribute their own successes to internal factors, such as ability or effort, while putting the blame for failure on external factors such as luck. This is the selfserving bias and suggests that feedback provided to employees will be distorted by recipients. 4. How does selectivity affect perception? Give an example of how selectivity can create perceptual distortion. Answer Selective perceptionAny characteristic that makes a person, object, or event stand out will increase the probability that it will be perceived. It is impossible for us to assimilate everything we see only certain stimuli can be taken in. Selectivity works as a shortcut in judging other people by allowing us to speed-read others, but not without the risk of drawing an inaccurate picture. Becaus e we see what we want to see, we can draw unwarranted conclusions from an ambiguous situation. 5. What is stereotyping? Give an example of how stereotyping can create perceptual distortion. Answer Stereotypingjudging someone on the basis of our perception of the group to which he or she belongs. Generalization is not without advantages. It is a means of simplifying a complex world, and it permits us to maintain consistency. The problem, of course, is when we inaccurately stereotype. From a perceptual standpoint, if people expect to see these stereotypes, that is what they will perceive, whether they are accurate or not. One of the problems of stereotypes is that they are widespread. 6. Give some positive results of using shortcuts when judging others. Answer They save us time and they help us process overloads of information effectively, but the drawbacks may out weigh such advantages. Employment interviewEvidence indicates that interviewers make perceptual judgments that are often inaccurate. Because interviews usually have so little consistent structure and interviewers vary in terms of what they are looking for in a candidate, judgments of the same candidate can vary widely. Performance expectationsEvidence demonstrates that people will attempt to validate their perceptions of reality, even when those perceptions are faulty. Performance evaluation An employees performance appraisal is very much dependent on the perceptual process. Employee effort An individuals future in an organization is usually not dependent on performance alone. An assessment of an individuals effort is a subjective judgment susceptible to perceptual distortions and bias. 7. What is the rational decision-making model? Under what conditions is it applicable? Answer The optimizing decision maker is rational. He/she makes consistent, value-maximizing choices within specified constraints. The Rational Modelsix steps listed in Exhibit 5-4. Define the problem. Identify the decision criteria important to solving the problem. Weight the previously identified criteria in order to give them the correct priority in the decision. Generate possible alternatives that could succeed in resolving the problem. Critically analyze and evaluate each alternative. Compute the optimal decision. Evaluate each alternative against the weighted criteria, and select the alternative with the highest total score.

8. Describe organizational factors that might constrain decision makers. Answer 1) The lack of problem clarityThe decision maker is assumed to have complete information regarding the decision situation. 2) Not knowing all the options It is assumed the decision maker is aware of all the possible consequences of each alternative. 3) Not being able to make clear preferences Criteria and alternatives can be ranked and weighted to reflect their importance. 4) Constant preferences When specific decision criteria are constant and the weights assigned to them are stable over time. 5) If there are important time or cost constraintsThe rational decision maker can obtain full information about criteria and alternatives because it is assumed that there are no time or cost constraints. 6) When there is no maximum payoff alternativeThe rational decision maker will choose the alternative that yields the highest perceived value. Other organizational constraints: Performance evaluation Managers are strongly influenced in their decision making by the criteria by which they are evaluated. Managers blocking negative information Reward systems The organizations reward system influences decision makers by suggesting to them what choices are preferable in terms of personal payoff. Programmed routines By programming decisions, organizations are able to get individuals to achieve high levels of performance without paying for the years of experience. System-imposed time constraints Organizations impose deadlines on decisions. Decisions must be made quickly in order to stay ahead of the competition and keep customers satisfied. Almost all important decisions come with explicit deadlines. Historical precedents Decisions have a context. Individual decisions are more accurately characterized as points in a stream of decisions. Decisions made in the past are ghosts which continually haunt current choices. It is common knowledge that the largest determining factor of the size of any given years budget is last years budget. 9. What role does intuition play in effective decision making? Answer Intuitive decision making has recently come out of the closet and into some respectability. We define intuitive decision making as an unconscious process created out of distilled experience. It operates in complement with rational analysis. The experts experience allows him or her to recognize the pattern in a situation and draw upon previously learned information associated with that pattern to quickly arrive at a decision choice. The result is that the intuitive decision maker can decide rapidly with what appears to be very limited information. Eight conditions when people are most likely to use intuitive decision making: When a high level of uncertainty exists When there is little precedent to draw on When variables are less scientifically predictable When facts are limited When facts do not clearly point the way to go When analytical data are of little use When there are several plausible alternative solutions to choose from, with good arguments for each When time is limited and there is pressure to come up with the right decis ion

10. Are unethical decisions more a function of the individual decision maker or the decision makers work environment? Explain. Answer This is an opinion question. In many ways, it parallels earlier questions whether heredity or environment shape personality. QUESTIONS FOR CRITICAL THINKING 1. How might the differences in experiences of students and instructors affect their perceptions of students written work and class comments? Answer When an individual looks at a target and attempts to interpret what he or she sees, that interpretation is heavily influenced by personal characteristics of the individual perceiver. The more relevant personal characteristics affecting perception are attitudes, motives, interests, past experiences, and expectations. Unsatisfied needs or motives stimulate individuals and may exert a strong influence on their perceptions. As interests narrow ones focus, so do ones past experiences. You perceive those things to which you can relate. Finally, expectations can distort your perceptions in that you will see what you expect to see. 2. An employee does an unsatisfactory job on an assigned project. Explain the attribution process that this persons manager will use to form judgments about this employees job performance. Answer Attribution theory suggests that when we observe an individuals behavior, we attempt to determine whether it was internally or externally caused. That determination depends largely on three factors: distinctiveness, consensus, and consistency. First, clarification of the differences between internal and external causation Internally caused behaviors are those that are believed to be under the personal control of the individual Externally caused behavior is seen as resulting from outside causes ; that is, the person is seen as having been forced into the behavior by the situation. Distinctiveness refers to whether an individual displays different behaviors in different situations. What we want to know is whether the observed behavior is unusual. If it is, the observer is likely to give the behavior an external attribution. If this action is not unusual, it will probably be judged as internal. If everyone who is faced with a similar situation responds in the same way, we can say the behavior shows consensus. If consensus is high, you would be expected to give an external attribution to the employees tardiness, whereas if other employees who took the same route made it to work on time, your conclusion as to causation would be internal. Consistency in a persons actions. Does the person respond the same way over time? The more consistent the behavior, the more the observer is inclined to attribute it to internal causes. Exhibit 5-3 summarizes the key elements in attribution theory. 3. For the most part, individual decision making in organizations is an irrational process. Do you agree or disagree? Discuss. Answer Students may argue either side, but they need to understand bounded rationality and the role of intuition in decision-making. When decision makers are faced with a simple problem having few alternative courses of action, and when the cost of searching out and evaluating alternatives is low, the rational model

is fairly accurate. Decision makers generally make limited use of their creativity. Choices tend to be confined to the neighborhood of the problem symptom and to the neighborhood of the current alternative. When faced with a complex problem, most people respond by reducing the problem to a level at which it can be readily understoodbounded rationality. The capacity of the human mind for formulating and solving complex problems is far too small to meet the requirements for full rationality. Intuitive decision making as an unconscious process created out of distilled experienceit operates in complement with rational analysis. The experts experience allows him or her to recognize the pattern in a situation and draw upon previously learned information associated with that pattern to quickly arrive at a decision choice. The result is that the intuitive decision maker can decide rapidly with what appears to be very limited information. 4. What factors do you think differentiate good decision makers from poor ones? Relate your answer to the sixstep rational model. Answer The optimizing decision maker is rational. He or she makes consistent, value-maximizing choices within specified constraints. The rational modelsix steps listed in Exhibit 5-4. Define the problemMany poor decisions can be traced to the decision maker overlooking a problem or defining the wrong problem. Identify the decision criteria important to solving the problem. This brings in the decision makers interests, values, and similar personal preferences. Any factors not identified in this step are considered irrelevant to the decision maker. Weight the previously identified criteria in order to give them the correct priority in the decision. Generate possible alternatives that could succeed in resolving the problem. Critically analyze and evaluate each alternative. Compute the optimal decision. 5. Have you ever increased your commitment to a failed course of action? If so, analyze the follow-up decision to increase your commitment, and explain why you behaved as you did. Answer Escalation of commitment is an increased commitment to a previous decision in spite of negative information. It has been well-documented that individuals escalate commitment to a failing course of action when they view themselves as responsible for the failure. Students analysis will vary with their experience. 1. Does motivation come from within a person, or is it a result of the situation? Explain. Answer Many people incorrectly view motivation as a personal trait that is, some have it and others do not. Motivation is the result of the interaction of the individual and the situation. The text defines motivation as the processes that account for an individuals intensity, direction, and persistence of effort toward attaining a goal. There are three key elements. Intensity is concerned with how hard a person tries. This is the element most of us focus on when we talk about motivation. Direction is the orientation that benefits the organization. Persistence is a measure of how long a person can maintain his/her effor t. Motivated individuals stay with a task long enough to achieve their goal.

2. What are the implications of Theories X and Y for motivation practices? Answer Douglas McGregor concluded that a managers view of the nature of human beings is based on a certain grouping of assumptions. Theory X assumptions are basically negative. Theory Y assumptions are basically positive. There are several implications. Theory X assumes that lower-order needs dominate individuals. Theory Y assumes that higher-order needs dominate individuals. McGregor himself held to the belief that Theory Y assumptions were more valid than Theory X. Either Theory X or Theory Y assumptions may be appropriate in a particular situation. 3. Compare and contrast Maslows hierarchy of needs theory with (a) Alderfers ERG theory and (b) Herzbergs two-factor theory. Answer Abraham Maslows hierarchy of needs. PhysiologicalIncludes hunger, thirst, shelter, sex, and other bodily needs SafetyIncludes security and protection from physical and emotional harm SocialIncludes affection, belongingness, acceptance, and friendship EsteemIncludes internal esteem factors such as self-respect, autonomy, and achievement; and also external esteem factors such as status, recognition, and attention Self-actualizationThe drive to become what one is capable of becoming; includes growth, achieving ones potential, and self-fulfillment See Exhibit 6-1. No need is ever fully gratified; a substantially satisfied need no longer motivates. Maslow separated the five needs into higher and lower orders. Physiological and safety needs were described as lower -order. Social, esteem, and self-actualization were described as higher-order needs. Higher-order needs are satisfied internally. Lower-order needs are predominantly satisfied externally. Two-factor theory is sometimes also called motivation-hygiene theory. It was proposed by psychologist Frederick Herzberg when he investigated the question, What do people want from their jobs? Herzberg concluded: Intrinsic factors, such as advancement, recognition, responsibility, and achievement seem to be related to job satisfaction. Dissatisfied respondents tended to cite extrinsic factors, such as supervision, pay, company policies, and working conditions. The opposite of satisfaction is not dissatisfaction. Removing dissatisfying characteristics from a job does not necessarily make the job satisfying. See Exhibit 6-3. Job satisfaction factors are separate and distinct from job dissatisfaction factors. To motivate people, emphasize factors that are intrinsically rewarding that are associated with the work itself or to outcomes directly derived from it. ERG theory. Clayton Alderfer reworked Maslows need hierarchy to align it with the empirical research. His revised need hierarchy is labeled ERG theory. Alderfer argues that there are three groups of core needs existence, relatedness, and growth. (cont.) The existence group Providing our basic material existence requirements. They include Maslows physiolo gical and safety needs. Relatedness The desire we have for maintaining important interpersonal relationships. These social and status desires

require interaction with others. They align with Maslows social need and the external component. Growth needs An intrinsic desire for personal development. These include the intrinsic component from Maslows esteem category and the characteristics included under self-actualization. Alderfers ERG theory also differs from Maslows in that: More than one need may be operative at the same time. If the gratification of a higher-level need is stifled, the desire to satisfy a lower-level need increases. ERG theory does not assume that there exists a rigid hierarchy. A person can be working on growth even though existence or relatedness needs are unsatisfied, or all three need categories could be operating at the same time. ERG theory also contains a frustration-regression dimension. Maslow argued that an individual would stay at a certain need level until that need was satisfied. ERG theory notes that when a higher-order need level is frustrated, the individuals desire to increase a lower-level need takes place. ERG theory is more consistent with our knowledge of individual differences among people. 4. Describe the three needs isolated by McClelland. How are they related to worker behavior? Answer Developed by David McClelland and his associates, the theory focuses on three needs: achievement, power, and affiliation. Need for achievement: The drive to excel, to achieve in relation to a set of standards, to strive to succeed Need for power: The need to make others behave in a way that they would not have behaved otherwise Need for affiliation: The desire for friendly and close interpersonal relationships Some people have a compelling drive to succeed. They are striving for personal achievement rather than the rewards of success, per se. This drive is the achievement need (nAch). McClelland found that high achievers differentiate themselves from others by their desire to do things better. The need for power (nPow) is the desire to have impact, to be influential, and to control others. Individuals high in nPow enjoy being in charge and strive for influence over others. They prefer to be placed into competitive and status-oriented situations, and tend to be more concerned with prestige and gaining influence over others than with effective performance. The third need isolated by McClelland is affiliation (nAfl). This need has received the least attention from researchers. Individuals with a high affiliation motive strive for friendship. Prefer cooperative situations rather than competitive ones, and desire relationships involving a high degree of mutual understanding. 5. Explain cognitive evaluation theory. How applicable is it to management practice? Answer Cognitive evaluation theory is well researched and supported. It suggests that when extrinsic rewards are used by organizations as payoffs for superior performance, the intrinsic rewards, which are derived from individuals doing what they like, are reduced. If the cognitive evaluation theory is valid, it should have major implications for managerial practices. If pay or other extrinsic rewards are to be effective motivators, they should be made c ontingent on an individuals performance. Cognitive evaluation theorists would argue that this will tend only to decrease the internal satisfaction that the individual receives from doing the job. If correct, it would make sense to make an individuals pay non-contingent on performance in order to avoid decreasing intrinsic motivation.

6. What is the role of self-efficacy in goal setting? Answer If factors such as ability and acceptance of the goals are held constant, we can also state that the more difficult the goal, the higher the level of performance. People will do better when they get feedback on how well they are progressing toward their goals. Self-generated feedback is more powerful a motivator than externally generated feedback. Self-efficacy refers to an individuals belief that he or she is capable of performing a task. The higher ones self-efficacy, the more confidence one has in his or her ability to succeed in a task. 7. Contrast distributive and procedural justice. What implications might they have for designing pay systems in different countries? Answer Historically, equity theory focused on distributive justice or the perceived fairness of the amount and allocation of rewards among individuals. Equity should also cons ider procedural justice, the perceived fairness of the process used to determine the distribution of rewards. The evidence indicates that distributive justice has a greater influence on employee satisfaction than procedural justice. Procedural justice tends to affect an employees organizational commitment, trust in his or her boss, and intention to quit. By increasing the perception of procedural fairness, employees are likely to view their bosses and the organization as positive even if they are dissatisfied with pay, promotions, and other personal outcomes. 8. Identify the variables in expectancy theory. Answer Expectancy theory is one of the most widely accepted explanations of motivation. It argues that the strength of a tendency to act in a certain way depends on the strength of an expectation that the act will be followed by a given outcome and on the attractiveness of that outcome to the individual. It says that an employee will be motivated to exert a high level of effort when he/she believes that: Effort will lead to a good performance appraisal. That a good appraisal will lead to organizational rewards. That the rewards will satisfy the employees personal goals. Three key relationships (See Exhibit 6-7.) Effort-performance relationship: the probability perceived by the individual that exerting a given amount of effort will lead to performance Performance-reward relationship: the degree to which the individual believes that performing at a particular level will lead to the attainment of a desired outcome Rewards-personal goals relationship: the degree to which organizational rewards satisfy an individuals personal goals or needs and the attractiveness of those potential rewards for the individual 9. Explain the formula: Performance = f(A M O), and give an example. Answer Success on a job is facilitated or hindered by the existence or absence of support resources. A popular although arguably simplistic way of thinking about employee performance is as a function of the interaction of ability and motivation; that is, performance = f(A M). If either is inadequate, performance will be negatively affected. We need to add opportunity to perform to our equation performance = f(A M O). See Exhibit 6-8. When you attempt to assess why an employee may not be performing to the level that you believe he or she is capable of, take a look at the work environment to see if it is supportive.

10. What consistencies among motivation concepts, if any, apply cross-culturally? Answer Exhibit 6-9 presents a model that integrates much of what we know about motivation. There are cross-cultural consistencies. The desire for interesting work seems important to almost all workers. Growth, achievement, and responsibility were rated the top three and had identical rankings in another study of several countries. QUESTIONS FOR CRITICAL THINKING 1. The cognitive evaluation theory is contradictory to reinforcement and expectancy theories. Do you agree or disagree? Explain. Answer As students make their arguments they should include the following facts from the text. Cognitive evaluation theory When extrinsic rewards are used by organizations as payoffs for superior performance, the intrinsic rewards, which are derived from individuals doing what they like, are reduced. The elimination of extrinsic rewards can produce a shiftfrom an external to an internal explanationin an individuals perception of causation of why he or she works on a task. If the cognitive evaluation theory is valid, it should have major implications for managerial practices. The theory may have limited applicability to work organizations because most low-level jobs are not inherently satisfying enough to foster high intrinsic interest, and many managerial and professional positions offer intrinsic rewards. Reinforcement theory Reinforcement theory is behavioristic in approach and is clearly in contradiction to cognitive evaluation theory. It argues that reinforcement conditions behavior. Reinforcement theorists see behavior as being environmentally caused. Reinforcement theory ignores the inner state of the individual and concentrates solely on what happens to a person when he or she takes some action. Reinforcement is undoubtedly an important influence on behavior, but few scholars are prepared to argue that it is the only influence. Expectancy theory Expectancy theory is one of the most widely accepted explanations of motivation. Expectancy theory argues that the strength of a tendency to act in a certain way depends on the strength of an expectation that the act will be followed by a given outcome and on the attractiveness of that outcome to the individual. It says that an employee will be motivated to exert a high level of effort when he/she believes that: Effort will lead to a good performance appraisal. That a good appraisal will lead to organizational rewards. That the rewards will satisfy the employees personal goals. Three key relationships (See Exhibit 6-7.) Effort-performance relationship: the probability perceived by the individual that exerting a given amount of effort will lead to performance Performance-reward relationship: the degree to which the individual believes that performing at a particular level will lead to the attainment of a desired outcome Rewards-personal goals relationship: the degree to which organizational rewards satisfy an individuals personal goals or needs and the attractiveness of those potential rewards for the individual The key to expectancy theory is the understanding of an individuals goals and the linkage between effort and performance, between performance and rewards, and finally, between the rewards and individual goal satisfaction. As a contingency model, expectancy theory recognizes that there is no universal principle for explaining everyones motivations. 2. Managers should be able, through proper selection and job design, to have every employee experience flow in his or her job. Do you agree or disagree? Discuss. Answer That would be a lofty accomplishment, however studies done on this theory have been done on management level employees and it remains to be seen whether or not it would apply to the rank and file

worker. However, managers can try to design jobs with the goal of increased job satisfaction using intrinsic factors as motivators. Thomas suggests that the are four components to be considered. They are: Choice: the ability to select tasks activities that make sent to you and perform them as you deem appropriate. Competence: the accomplishment you feel in skillfully performing tasks activities you have chosen. Meaningfulness: the opportunity to pursue a worthy task purposethat matters in the larger scheme of things. Progress: feeling you are making significant advancement in achieving the tasks purpose. Jobs requiring high levels of education, training, and skill with a high degree of autonomy would be the most likely to allow employees to experience the flow. 3. Analyze the application of Maslows and Herzbergs theories to an African or Caribbean nation where more than a quarter of the population is unemployed. Answer Students may lack sufficient knowledge of these cultures to fully answer the question. They should focus on the issues of large scale unemployment. It should be relatively obvious to students that people in other cultures world rank the need categories differently and that what is satisfaction or motivation will vary as well. Abraham Maslows hierarchy of needs. PhysiologicalIncludes hunger, thirst, shelter, sex, and other bodily needs SafetyIncludes security and protection from physical and emotional harm SocialIncludes affection, belongingness, acceptance, and friendship EsteemIncludes internal esteem factors such as self-respect, autonomy, and achievement; and external esteem factors such as status, recognition, and attention Self-actualizationThe drive to become what one is capable of becoming; includes growth, achieving ones potential, and self-fulfillment See Exhibit 6-1. Motivation-hygiene theory Intrinsic factors, such as advancement, recognition, responsibility, and achievement seem to be related to job satisfaction. Dissatisfied respondents tended to cite extrinsic factors, such as supervision, pay, company policies, and working conditions. Job satisfaction factors are separate and distinct from job dissatisfaction factors. 4. Can an individual be too motivated, so that his or her performance declines as a result of excessive effort? Discuss. Answer Students responses may vary but should consider the following elements from the text. McClelland found that high achievers differentiate themselves from others by their desire to do things better. If a high achiever sets too high goals for him/herself, he/she could end up demotivating him/herself because of the failure to achieve the goals. According to cognitive evaluation theory, when extrinsic rewards are used by organizations as payoffs for superior performance, the intrinsic rewards, which are derived from individuals doing what they like, are reduced. The popular explanation is that the individual experiences a loss of control over his/her own behavior so that the previous intrinsic motivation diminishes. Furthermore, the elimination of extrinsic rewards can produce a shiftfrom an external to an internal explanationin an individuals perception of causation of why he/she works on a task.

5. Identify three activities you really enjoy (for example, playing tennis, reading a novel, going shopping). Next, identify three activities you really dislike (for example, going to the dentist, cleaning the house, staying on a restricted-calorie diet). Using the expectancy model, analyze each of your answers to assess why some activities stimulate your effort while others do not. Answer Students responses will vary but should take into consideration the following elements of expectancy theory. The strength of a tendency to act in a certain way depends on the strength of an expectation that the act will be followed by a given outcome and on the attractiveness of that outcome to the individual. It says that an employee will be motivated to exert a high level of effort when he/she believes that: Effort will lead to a good performance appraisal. That a good appraisal will lead to organizational rewards. That the rewards will satisfy the employees personal goals. Three key relationships Effort-performance relationship: the probability perceived by the individual that exerting a given amount of effort will lead to performance Performance-reward relationship: the degree to which the individual believes that performing at a particular level will lead to the attainment of a desired outcome Rewards-personal goals relationship: the degree to which organizati onal rewards satisfy an individuals personal goals or needs and the attractiveness of those potential rewards for the individual

1. Relate goal-setting theory to the MBO process. How are they similar? Different? Answer Management by objectives emphasizes participatively set goals that are tangible, verifiable, and measurable. It is not a new idea. Linking MBO and goal-setting theory: Goal-setting theory demonstrates that: Hard goals result in a higher level of individual performance than do easy goals . Specific hard goals result in higher levels of performance than no goals at all or generalized goals. Feedback on ones performance leads to higher performance. MBO Directly advocates specific goals and feedback. Implies that goals must be perceived as feasible. Is most effective when the goals are difficult enough to require stretching. The only area of possible disagreement is participation. MBO strongly advocates it. Goal-setting theoryassigning goals to subordinates frequently works just as well as participation. 2. What is an ESOP? How might it positively influence employee motivation? Answer ESOPs are company-established benefit plans in which employees acquire stock as part of their benefits. In the typical ESOP, an employee stock ownership trust is created. Companies contribute either stock or cash to buy stock for the trust and allocate the stock to employees. Employees usually cannot take physical possession of their shares or sell them as long as they are still employed at the company. The research indicates that ESOPs increase employee satisfaction, but their impact on performance is less clear. One study compared 45 ESOPs against conventional companies. The ESOPs outperformed the conventional firms both in terms of employment and sales growth, but other studies have shown disappointing results. The evidence consistently indicates that it takes ownership and a participative style of management to achieve significant improvements in an organizations performance.

3. Explain the roles of employees and management in quality circles. Answer Probably the most widely discussed and undertaken formal style of employee involvement is management in quality circles. It is a work group of eight to ten employees and supervisors who have a shared area of responsibility. They meet regularly on company time to discuss their quality problems, investigate causes of the problems, recommend solutions, and take corrective actions. They take over the responsibility for solving quality problems, and they generate and evaluate their own feedback. Management typically retains control over the final implementation decision. Exhibit 7-3 describes a typical quality circle process. 4. What are the pluses of variable-pay programs from an employees viewpoint? From managements viewpoint? Answer Variable pay offers bigger rewards and ties rewards to work. For management, it reduces cost because pay is tied to productivity and eliminates or reduces permanent increases in labor costs. 5. Contrast job-based and skill-based pay. Answer Rather than job title defining ones pay (as in job-based pay), ones competency (or the number and level of skills one has) determines pay levels. 6. What is gain sharing? What explains its recent popularity? Answer This is a formula-based group incentive plan. Improvements in group productivityfrom one period to anotherdetermine the money allocated. Gain sharing and profit sharing are similar but not the same thing. Gainsharing rewards specific behaviors that are less influenced by external factors. Employees in a gainsharing plan can receive incentive awards even when the organization is not profitable. Studies generally support that organizations with profit-sharing plans have higher levels of profitability than those without. Gain sharing has been found to improve productivity in a majority of cases and often has a positive impact on employee attitudes. 7. What motivates professional employees? Answer The typical employee today is more likely to be a highly trained professional with a college degree than a blue-collar factory worker. They receive a great deal of intrinsic satisfaction from their work. They tend to be well paid. What motivates professionals? Money and promotions typically are low on their priority list. Job challenge tends to be ranked high. They like to tackle problems and find solutions. Provide them with ongoing challenging projects. Give them autonomy to follow their interests, and allow them to structure their work. Reward them with educational opportunities, and also reward them with recognition. An increasing number of companies are creating alternative career paths for their professional and technical people, allowing employees to earn more money and status, without assuming managerial responsibilities. 8. What motivates contingent employees? Answer In 1995, approximately six million Americans, or 4.9 percent of those with jobs, considered themselves as part of the contingent workforce. Temporary workers also are typically provided with little or no health care, pensions, or similar benefits. There is no simple solution for motivating temporary employees. What will motivate involuntarily temporary employees? An opportunity for permanent status. The opportunity for training. From an equity standpoint, you should also consider the repercussions of mixing permanent and temporary workers where pay differentials are significant.

9. Is it possible to motivate low-skilled service workers? Discuss. Answer It is one of the most challenging motivation problems in industries such as retailing and fast food. Traditional approaches for motivating these people have focused on providing more flexible work schedules, and filling these jobs with teenagers and retirees whose financial needs are less has met with less than enthusiastic results. Unless pay and benefits are significantly increased, high turnover probably has to be expected in these jobs. This can be somewhat offset by widening the recruiting net, making these jobs more appealing, and raising pay levels. 10. What can you do, as a manager, to increase the likelihood that your employees will exert a high level of effort? Answer Check the system for equity. Rewards should also be perceived by employees as equating with the inputs they bring to the job. At a simplistic level, this should mean that experience, skills, abilities, effort, and other obvious inputs should explain differences in performance and, hence, pay, job assignments, and other obvious rewards. QUESTIONS FOR CRITICAL THINKING 1. Identify five different criteria by which organizations can compensate employees. Based on your knowledge and experience, do you think performance is the criterion most used in practice? Discuss. Answer Seniority, position, merit, skill, and productivity/perf ormance. Students responses will vary; generally, there is a great deal of discussion of performance pay in the literature and among managers. The key issue is whether employees perceive the compensation-performance linkage, regardless of what management implements. 2. Recognition may be motivational for the moment but it does not have any staying power. It is an empty reinforcer. Why? Because when you go the grocery store, they do not take recognition as a form of payment! Do you agree or disagree? Discuss. Answer Students answers may vary more due to the personalities than facts. Some people never forget being recognized once, others look for continual recognition. Keys to remember in the discussion include the following: The best recognition programs use multiple sources and recognize both individual and group accomplishments. Employees consider recognition to be the most powerful workplace motivator. Consistent with reinforcement theory, rewarding a behavior with recognition immediately following that behavior is likely to encourage its repetition. 3. Performance cannot be measured, so any effort to link pay with performance is a fantasy. Differences in performance are often caused by the system, which means the organization ends up rewarding the circumstances. It is the same thing as rewarding the weather forecaster for a pleasant day. Do you agree or disagree with this statement? Support your position. Answer Performance measurement can be difficult depending on the type of job being evaluated. The key is to try to control for those factors that employees cannot control, measure, and reward those that employees

can control. Also, building in base income in an incentive system helps smooth the dips that come due to factors beyond employee control. 4. It is an indisputable fact that there has been an explosive increase in the difference between the average U.S. workers income and those of senior executives. In 1980, the average CEO made 42 times the average bluecollar workers pay. In 1990, it was 85 times. In 2000, it had risen to 531 times. What are the implications of this trend for motivation in organizations? Answer Students will have varying opinions on this. Given recent corporate scandals involving CEOs who took millions at the expense of employees retirement accounts they should take into account such factors as trust, employee retention, productivity, efficiency, etc. 5. Your text argues for recognizing individual differences. It also suggests paying attention to members of diversity groups. Is this contradictory? Discuss. Answer There is no inherent contradiction because both approaches seek to treat people as individuals. Sensitivity to diversity is not sensitivity to a group, but to the individual within the group. 1. Compare and contrast command, task, interest, and friendship groups. Answer A group is defined as two or more individuals, interacting and interdependent, who have come together to achieve particular objectives. Groups can be either formal or informal. It is possible to subclassify groups as command, task, interest, or friendship groups. A command group is determined by the organization chart. It is composed of direct reports to a given manager. Task groupsorganizationally determined, represent those working together to complete a job task. A task groups boundaries are not limited to its immediate hierarchical superior. It can cross command relationships. For instance, if a college student is accused of a campus crime, it may require communication and coordination among the dean of academic affairs, the dean of students, the registrar, the director of security, and the students advisor. All command groups are also task groups, but the reverse need not be true. An interest group is people who affiliate to attain a specific objective with which each is concerned. Employees who band together to have their vacation schedules altered. Friendship groups often develop because the individual members have one or more common characteristics. Social alliances, which frequently extend outside the work situation, can be based on similar age or ethnic heritage. 2. What might motivate you to join a group? Answer Informal groups satisfy their members social needs. There is no single reason why indi viduals join groups. Exhibit 8-1 summarizes the most popular reasons people have for joining groups. 3. Describe the five-stage group-development model. Answer Exhibit 8-2 shows the five-stage group-development model: The first stage is forming. Characterized by a great deal of uncertainty about the groups purpose, structure, and leadership. Members are trying to determine what types of behavior are acceptable. Stage is complete when members have begun to think of themselves as part of a group. The second stage is storming. One of intragroup conflict Members accept the existence of the group, but there is resistance to constraints on individuality. There is conflict over who will control the group When complete, there will be a relatively clear hierarchy of leadership within the group. The third stage is norming. One in which close relationships develop and the group demonstrates cohesiveness.

There is now a strong sense of group identity and camaraderie. The stage is complete when the group structure solidifies and the group has assimilated a common set of expectations of what defines correct member behavior. The fourth stage is performing. The structure at this point is fully functional and accepted. Group energy has moved from getting to know and understand each other to performing. 4. What is the relationship between a work group and the organization of which it is a part? Answer An organizations overall strategy, typically put into place by top management, outlines the organizations goals and the means for attaining these goals. The strategy will influence the power of various work groups, which will determine the resources that the organizations top management is willing to allocate to it for performing its tasks. Organizations have authority structures that define who reports to whom, who makes decisions, and what decisions individuals or groups are empowered to make. Organizations create rules, procedures, policies, job descriptions, and other forms of formal regulations to standardize employee behavior. The presence or absence of resources such as money, time, raw materials, and equipment which are allocated to the group by the organizationhave a large bearing on the groups behavior. The criteria that an organization uses in its selection process will determine the kinds of people that will be in its work groups. The performance evaluation and reward system. Group members behavior will be influenced by how the organization evaluates performance and what behaviors are rewarded. Every organization has an unwritten culture that defines standards of acceptable and unacceptable behavior for employees. Members of work groups have to accept the standards implied in the organizations dominant culture if they are to remain in good standing. The physical work setting creates both barriers and opportunities for work group interaction. 5. What are the implications of Zimbardos prison experiment for OB? Answer The simulation actually proved too successful in demonstrating how quickly individuals learn new roles. The researchers had to stop the experiment after only six days because of the pathological reactions that the participants were demonstrating. The participants had learned stereotyped conceptions of guard and prisoner roles from the mass media and their own personal experiences in power and powerless relationships at home. This allowed them easily and rapidly to assume roles that were very different from their inherent personalities. 6. Explain the implications from the Asch experiments. Answer The results obtained by Asch demonstrated that over many experiments and many trials, subjects conformed in about 35 percent of the trials; the subjects gave answers that they knew were wrong but that were consistent with the replies of other group members. There have been changes in the level of conformity over time. Levels of conformity have steadily declined. Aschs findings are culture-bound. Conformity to social norms is higher in collectivist cultures than in individualistic cultures.

7. How are status and norms related? Answer All groups have normsacceptable standards of behavior that are shared by the groups members. Norms tell members what they ought and ought not to do under certain circumstances. Norms are important because they: Facilitate the groups survival. Increase the predictability of group members behaviors. Reduce embarrassing interpersonal problems for group members. Allow members to express the central values of the group and clarify what is distincti ve about the groups identity. There is considerable evidence that groups can place strong pressures on individual members to change their attitudes and behaviors to conform to the groups standard. Status is a socially defined position or rank given to groups or group members by others. We live in a classstructured society despite all attempts to make it more egalitarian. High-status members of groups often are given more freedom to deviate from norms than other group members. High-status people also are better able to resist conformity pressures. The previous findings explain why many star athletes, famous actors, topperforming salespeople, and outstanding academics seem oblivious to appearance or social norms. 8. When do groups make better decisions than individuals? Answer The answer is, it depends. Groups are more effective in terms of accuracy and often make better quality decisions than the individual. They also tend to have more creative decisions. However, in terms of speed and efficiency, individuals are more effective. 9. How can a groups demography help you to predict turnover? Answer An offshoot of the composition issue is the degree to which members of a group share a common demographic attribute and the impact of this attribute on turnover. The individuals attribute is in relationship to the attributes of others with whom he/she works. Groups and organizations are composed of cohorts, which we define as individuals who hold a common attribute. Group demography should help us to predict turnover. Turnover will be greater among those with dissimilar experiences because communication is more difficult. Conflict and power struggles are more likely, and more severe when they occur. This makes group membership less attractive, so employees are more likely to quit. The implication is that the composition of a group may be an important predictor of turnover. 10. What is groupthink? What is its effect on decision-making quality? Answer Groupthink describes situations in which group pressures for conformity deter the group from critically appraising unusual, minority, or unpopular views. The phenomenon that occurs when group members become so enamored of seeking concurrence that the norm for consensus overrides the realistic appraisal of alternative courses of action and the full expression of deviant, minority, or unpopular views. It is a deterioration in an individuals mental efficiency, reality testing, and moral judgment as a result of group pressures. Group members rationalize any resistance to the assumptions they have made. Members apply

direct pressures on those who momentarily express doubts. Those members who hold differing points of view seek to avoid deviating from group consensus by keeping silent. There appears to be an illusion of unanimity. In studies of historic American foreign policy decisions, these symptoms were found to prevail when government policy-making groups failed. Groupthink appears to be closely aligned with the conclusions Asch drew from his experiments. Groupthink does not attack all groups. It occurs most often where there is a clear group identity, where members hold a positive image of their group which they want to protect, and where the group perceives a collective threat to this positive image. QUESTIONS FOR CRITICAL THINKING 1. How could you use the punctuated-equilibrium model to better understand group behavior? Answer Temporary groups with deadlines do not seem to follow the traditional model. Their pattern is called the punctuated-equilibrium model. Shown in Exhibit 8-3. Studies indicate their own unique sequencing. The punctuated-equilibrium model characterizes groups as exhibiting long periods of inertia interspersed with brief revolutionary changes triggered primarily by their members awareness of time and deadlines. 2. Identify five roles you play. What behaviors do they require? Are any of these roles in conflict? If so, in what way? How do you resolve these conflicts? Answer Students answers will vary. Some suggested roles: student, sibling, child, adult, group leader, member of a social group, etc. Behaviors and conflicts will vary with role. 3. High cohesiveness in a group leads to higher group productivity. Do you agree or disagree ? Explain. Answer Groups differ in their cohesivenessthe degree to which members are attracted to each other and are motivated to stay in the group. Cohesiveness is important because it has been found to be related to the groups productivity. The relationship of cohesiveness and productivity depends on the performancerelated norms established by the group. If performance-related norms are high, a cohesive group will be more productive, but if cohesiveness is high and performance norms are low, productivity will be low. See Exhibit 8-7. Students responses will vary based on their perception and integration of the above facts. 3. What effect, if any, do you expect that workforce diversity has on performance and satisfaction? Answer Research studies generally substantiate that heterogeneous groups those composed of dissimilar individualsare more likely to have diverse abilities and information and should be more effective, especially on cognitive, creativity-demanding tasks. The group may be more conflict laden and less expedient. Essentially, diversity promotes conflict, which stimulates creativity, which leads to improved decision making. Diversity created by racial or national differences interfere with group processes, at least in the short term. Cultural diversity seems to be an asset on tasks that call for a variety of viewpoints. Such groups have more difficulty in learning to work with each other and solving problems. These difficulties seem to dissipate with time as it takes time for diverse groups to learn how to work through disagreements and different approaches to solving problems.

5. If group decisions consistently achieve better quality outcomes than those achieved by individuals, how did the phrase a camel is a horse designed by a committee become so popular and ingrained in the culture? Answer Students responses will vary. Generally, two main factors may have contributed to this mythology. The first is that individuals may blame the group for poor decisions, when in fact the decision was the result of a dominant member of the group. Second, there have been some colossal public screw-ups attributable to group decisions and the groupthink phenomenon. 1. Contrast self-managed and cross-functional teams. Answer Problem-solving teams did not go far enough in getting employees involved in work-related decisions and processes. These groups of employees (typically 10 15 in number) perform highly related or interdependent jobs and take on many of the responsibilities of their former supervisors. This includes planning and scheduling of work, assigning tasks to members, collective control over the pace of work, making operating decisions, and taking action on problems. Fully self-managed work teams even select their own members and have the members evaluate each others performance. Cross-functional teams are teams made up of employees from about the same hierarchical level, but from different work areas, who come together to accomplish a task. Many organizations have used horizontal, boundary-spanning groups for years. A task force is really nothing other than a temporary crossfunctional team. The popularity of cross-discipline work teams exploded in the late 1980s. All the major automobile manufacturers have turned to these forms of teams in order to coordinate complex projects. 2. Contrast virtual and face-to-face teams. Answer Virtual teams use computer technology to tie together physically dispersed members in order to achieve a common goal. The three primary factors that differentiate virtual teams are: The absence of paraverbal and nonverbal cues. These help clarify communication by providing increased meaning, but are not available in online interactions. Limited social context. Virtual teams often suffer from less social rapport and less direct interaction among members. The ability to overcome time and space constraints. Virtual teams allow people to work together who might otherwise never be able to collaborate. 3. List and describe nine team roles. Answer Nine potential team roles. See Exhibit 9-5. 4. How do effective teams minimize social loafing? Answer Effective teams undermine this tendency by holding themselves accountable at both the individual and team level. 5. How do effective teams minimize groupthink? Answer Conflict on a team is not necessarily bad. Teams that are completely void of conflict are likely to become apathetic and stagnant. It is often beneficial because it lessens the likelihood of groupthink. Effective teams will be characterized by an appropriate level of conflict.

6. List and describe the process variables associated with effective team performance. Answer These include member commitment to a common purpose, establishment of specific team goals, team efficacy, a managed level of conflict, and minimizing social loafing. A common purpose Effective teams have a common and meaningful purpose that provides direction, momentum, and commitment for members. This purpose is a vision. It is broader than specific goals. Specific goals Successful teams translate their common purpose into specific, measurable, and realistic performance goals. Specific goals facilitate clear communication and help teams maintain their focus on results. Team goals should be challenging. Team efficacy Effective teams have confidence in themselves and believe they can succeedthis is team efficacy. Success breeds success. Management can increase team efficacy by helping the team to achieve small successes and skill training. Small successes build team confidence. The greater the abilities of team members, the greater the likelihood that the team will develop confidence and the capability to deliver on that confidence. Conflict levels Conflict on a team is not necessarily bad. Teams that are completely void of conflict are likely to become apathetic and stagnant. Relationship conflictsthose based on interpersonal incompatibilities, tension, and animosity toward othersare almost always dysfunctional. On teams performing nonroutine activities, disagreements among members about task content (called task conflicts) is not detrimental. It is often beneficial because it lessens the likelihood of groupthink. Effective teams will be characterized by an appropriate level of conflict. Social loafing Individuals can hide inside a group. Effective teams undermine this tendency by holding themselves accountable at both the individual and team level. 7. Under what conditions will the challenge of creating team players be greatest? Answer An employees success is no longer defined in terms of individual performance. To perform well as team members, individuals must be able to communicate openly and honestly, to confront differences and resolve conflicts, and to sublimate personal goals for the good of the team. The challenge of creating team players will be greatest where: The national culture is highly individualistic. The teams are being introduced into an established organization that has historically valued individual achievement. On the other hand, the challenge for management is less demanding when teams are introduced where employees have strong collectivist values. 8. What role do teams play in quality management? Answer The essence of TQM is process improvement, and employee involvement is the linchpin of process improvement. All such techniques and processes require high levels of communication and contact, response and adaptation, and coordination and sequencing. They require the environment that can be supplied only by superior work teams. Teams provide the natural vehicle for employees to share ideas and to implement improvements.

9. Contrast the pros and cons of having diverse teams. Answer Managing diversity on teams is a balancing act. See Exhibit 9-6. Diversity typically provides fresh perspectives on issues but it makes it more difficult to unify the team and reach agreements. The strongest case for diversity on work teams is when these teams are engaged in problem-solving and decisionmaking tasks. Heterogeneous teams bring multiple perspectives to the discussion, thus increasing the likelihood that the team will identify creative or unique solutions. Cohesiveness is likely to be lower on diverse teams. Diversity is detrimental to group cohesiveness. We suggest that if the norms of the team are supportive of diversity, then a team can maximize the value of heterogeneity while, at the same time, achieving the benefits of high cohesiveness. This makes a strong case for team members to participate in diversity training. 10. How can management invigorate stagnant teams? Answer Just because a team is performing well is no assurance that it will continue to do so. Effective teams can become stagnant. The text offers several suggestions for reinvigorating mature teams. Prepare members to deal with the problems of maturity. Offer refresher training. Offer advanced training. Encourage teams to treat their own development as a constant learning experience.

QUESTIONS FOR CRITICAL THINKING 1. Dont teams create conflict? Isnt conflict bad? Why, then, would management support the concept of teams? Answer Conflict on a team is not necessarily bad. Teams that are completely void of conflict are likely to become apathetic and stagnant. Relationship conflicts those based on interpersonal incompatibilities, tension, and animosity toward others-are almost always dysfunctional. On teams performing nonroutine activities, disagreements among members about task content (called task conflicts) is not detrimental. It is often beneficial because it lessens the likelihood of groupthink. Effective teams will be characterized by an appropriate level of conflict. Management can use selection, as some people already possess the interpersonal skills to be effective team players. Also, workshops help employees improve their problemsolving, communication, negotiation, conflict-management, and coaching skills. The reward system needs to encourage cooperative efforts rather than competitive ones. Promotions, pay raises, and other forms of recognition should be given to individuals for how effective they are as a collaborative team member. 2. Are there factors in the Japanese society that make teams more acceptable in the workplace than in the United States or Canada? Explain. Answer Japan is a more collectivist cultureteams have been in place for years. The United States and Canada are individualistic cultures. Social pressure is a major influence on all of life in Japan. It is not as much in the United States and Canada. 3. What problems might surface in teams at each stage in the five-stage group development model? Answer This question refers students back to the content in Chapter 8. See Exhibit 8-2 for the fivestage group-development model. The first stage is formingCharacterized by a great deal of uncertainty about the groups purpose, structure, and leadership. The storming stageOne of intragroup conflict. Conflict over who will control the group. The third stage is normingThere is now a strong sense of group identity and camaraderie.

Groupthink. The fourth stage is performingThe structure at this point is fully functional and accepted. For temporary committees, an adjourning stageAttention is directed toward wrapping up activities. Some may be depressed over the loss of camaraderie and friendships. 4. How do you think member expectation might affect team performance? Answer Since team performance is the sum or synergy of all members efforts, what each member expects of him/herself and his/her teammates will heavily influence output and cooperation. 5. Would you prefer to work alone or as part of a team? Why? How do you think your answer compares with others in your class? Answer This is an individual question, more of a discussion question. The key here is for the students to be honest with themselves and to become self-aware. It is not bad to want to be an individual contributor; it may mean that the student needs to pay particular attention to the culture of the company he/she goes to work for. 1. Describe the functions that communication provides within a group or organization. Give an example of each. Answer There are four major functions. Communication acts to control member behavior in several ways. When work groups tease or harass a member who produces too much, they are informally communicating with, and controlling, the members behavior. Communication fosters motivation by clarifying to employees what is to be done, how well they are doing, and what can be done to improve performance. The formation of specific goals, feedback on progress toward the goals, and reinforcement of desired behavior all stimulate motivation and require communication. Communication provides a release for the emotional expression of feelings and for fulfillment of social needs. For many employees, their work group is a primary source for social interaction. Communication facilitates decision-making. It provides the information. 2. Contrast encoding and decoding. Answer Encodingthe process of using the code or group of symbols to transfer meaning, the content of the message itself, and the decisions that we make in selecting and arranging both codes and content. The source initiates a message by encoding a thought. Four conditions have been described that affect the encoded message: Skillones total communicative success includes speaking, reading, listening, and reasoning skills. Attitudesinfluence our behavior. We hold predisposed ideas on numerous topics, and our communications are affected by these attitudes. Knowledgerestricts our communicative activity by the extent of our knowledge of the particular topic. Our position in the social-cultural system influences our behavior. Your beliefs and values, all part of your culture, act to influence you as a communicative source. Decodingthe symbols in the message must be translated into a form that can be understood by the receiver. The receiver is limited by his/her skills, attitudes, knowledge, and social-cultural system. 3. Contrast downward with upward communication. Answer Communication can flow vertically or laterally. Vertical communication can be either downward or upward. Downward communication flows from one level of a group or organization to a lower level. It is used to provide managers to assign goals, provide job instructions, inform employees of policies and procedures,

and offer feedback about performance. Upward communication flows to a higher level in the group or organization. It is used to provide feedback to higher ups, progress toward goals and relay current problems. It also keeps managers aware about how employees feel about their jobs. 4. What is nonverbal communication? Does it aid or hinder verbal communication? Answer Nonverbal communication involves body language, and paralinguistics describes the nonverbal aspects of communication that encompass tone of voice, pacing, pitch and similar aspects that go beyond the spoken word. It has been argued that every body movement has a meaning and that no movement is accidental. The two most important messages that body language conveys are the extent to which an individual likes another and is interested in his or her views and the relative perceived status between a sender and receiver. The specific meaning of any single body movement may be unclear; body language adds to and often complicates verbal communication. Body language differs between cultures. If you read the verbatim minutes of a meeting, you could not grasp the impact of what was said in the same way you could if you had been there or saw the meeting on video. It is important for the receiver to be alert to these nonverbal aspects of communication. Look for nonverbal cues as well as listen to the literal meaning of a senders words. Be aware of contradictions between the messages. We misinform others when we express one emotion verbally but nonverbally communicate a contradictory message. 5. What conditions stimulate the emergence of rumors? Answer Research indicates that rumors emerge as a response to situations that are important to us, where there is ambiguity, and under conditions that arouse anxiety. The grapevine is an important part of any group or organizations communication network and well worth understanding. 6. What are the advantages and disadvantages of e-mail? Answer Email uses the Internet to transmit and receive computer-generated text and documents. It has greatly reduced the number of memos, letters, and phone calls historically used to communicate among employees. Benefits include: they can be quickly written, edited and stored, they are easily distributed, they are a fraction of the cost of printed communications. Drawbacks include: information overload, the time to read excessive amounts of email, lack of emotional content. 7. What can managers do to improve their skills at providing performance feedback? Answer The following four suggestions help a manager be more effective when giving performance feedback. Focus on specific behaviors. Feedback should be specific rather than general. Keep it impersonal. Feedback, particularly the negative kind, should be descriptive rather than judgmental or evaluative. Make it well timed. Feedback is most meaningful when there is a short interval between behavior and feedback. If negative, make sure the behavior is controllable by the recipient. Negative feedback, therefore, should be directed toward the behavior the recipient can do something about. 8. What does the phrase sometimes the real message in a communication is buried in the silence mean? Answer Silencedefined as an absence of speech or noisecan be interpreted as an inaction or nonbehavior. Not necessarily, however, it can be a powerful form of communication. It can mean someone is thinking, is anxious and fearful of speaking, and it can signal disagreement, dissent, frustration, or anger.

Silence is a critical element of groupthink. It can also be a way for employees to express dissatisfaction and suffer in silence. Failing to pay close attention to silence can result in missing a vital part of the message. 9. Describe how political correctness can hinder effective communication. Answer Politically correct language is contributing a new barrier to effective communication. When we eliminate words from usage because they are politically incorrect, we reduce our options for conveying messages in the clearest and most accurate form. By removing certain words from our vocabulary, we make it harder to communicate accurately. We must be sensitive to how our choice of words might offend others. But we also have to be careful not to sanitize our language to the point where it clearly restricts clarity of communication. 10. Contrast high and low context cultures. Answer Cultures tend to differ in the importance to which context influences meaning. Countries like China, Vietnam, and Saudi Arabia are high-context cultures. They rely heavily on nonverbal and subtle situational cues when communicating with others. What is not said may be more significant than what is said. A persons official status, place in society, and reputation carry considerable weight. People from Europe and North America reflect their low-context cultures. They rely essentially on words to convey meaning. Body language or formal titles are secondary to spoken and written words. See Exhibit 10-9. Communication in high-context cultures implies considerably more trust by both parties. Oral agreements imply strong commitments in high-text cultures. Who you areyour age, seniority, rank in the organizationare highly valued and heavily influence your credibility. In low-context cultures, enforceable contracts will tend to be in writing, precisely worded, and highly legalistic. Similarly, low-context cultures value directness. QUESTIONS FOR CRITICAL THINKING 1. Ineffective communication is the fault of the sender. Do you agree or disagree? Discuss. Answer If students take a sender focus, they will agree. If they take an active listening posture, they will disagree. The source initiates a message by encoding a thought. Four conditions have been described that affect the encoded message: 1) skill, 2) attitudes, 3) knowledge, and 4) our position in the social-cultural system influence our behavior. The senders beliefs and values, all part of culture, act to influence the sender as a communicative source. 2. What can you do to improve the likelihood that your communiqus will be received and understood as you intend? Answer There are several things you can do. Use multiple channels / Tailor the message to the audience / Empathize with others Put yourself in their shoes / Practice active listening / Match your words and actions Remember the value of face-to-face communication when dealing with change. Use the grapevine / Use feedback As communicators, students need to also consider the following barriers to effective communication: Filtering refers to a sender manipulating information so that it will be seen more favorably by the receiver. Selective perceptionThe receivers in the communication process selectively see and hear based on their needs, motivations, experience, background, and other personal characteristics.

Information overloadResearch indicates that most of us have difficulty working with more than about seven pieces of information. When individuals have more information than they can sort out and use, they tend to select out, ignore, pass over, or forget information. DefensivenessEngaging in behaviors such as verbally attacking others, making sarcastic remarks, being overly judgmental, and questioning others motives LanguageWords mean different things to different people. The meanings of words are not in the words; they are in us. 3. How might managers use the grapevine for their benefit? Answer The grapevine is an important part of any group or organizations com munication network and well worth understanding. It identifies for managers those confusing issues that employees consider important and anxiety-provoking. It acts as both a filter and a feedback mechanism, picking up the issues that employees consider relevant. By assessing which liaison individuals will consider a given piece of information to be relevant, we can improve our ability to explain and predict the pattern of the grapevine. Management cannot eliminate rumors but they can minimize the negative consequences. Exhibit 10-5 offers a few suggestions for minimizing those negative consequences. 4. Using the concept of channel richness, give examples of messages best conveyed by e-mail, by faceto-face communication, and on the company bulletin board. Answer Recent research has found that channels differ in their capacity to convey information. Some are rich in that they have the ability to: 1) handle multiple cues simultaneously, 2) facilitate rapid feedback, 3) be very personal. Exhibit 10-6 illustrates that face-to-face talk scores highest in terms of channel richness. 5. Why do you think so many people are poor listeners? Answer It requires a focus on others. It requires effort; one has to pay attention. Few people are trained in active listening skills. 1. Trace the development of leadership research. Answer Leadership research began with trait research and goes back to the 1930s. Research efforts at isolating leadership traits resulted in a number of dead ends. A review of 20 different studies identified nearly 80 leadership traits, but only five of these traits were common to four or more of the investigations. The cumulative findings from a half of a century of research showed that some traits increase the likelihood of success as a leader, but none guarantee success. Leadership research from the late 1940s through the mid-1960s emphasized the preferred behavioral styles that leaders demonstrated. Researchers began to wonder if there was something unique in the way that effective leaders behave. Behavioral studies suggest that we could train people to be leaders. The most comprehensive and replicated of the behavioral theories resulted from research that began at Ohio State University in the late 1940s. University of Michigan leadership studies were undertaken at the same time as those being done at Ohio State, with similar research objectives. 2. Describe the strengths and weaknesses in the trait approach to leadership. Answer The trait approach has at least four limitations. First, there are no universal traits that predict in all situations. Second, traits predict behavior more in weak situations than in strong situations. Third, the

evidence is unclear in separating cause from effect. Finally, traits do a better job at predicting the appearance of leadership than in actually distinguishing between effective and ineffective leaders. These limitations have led researchers to look in other directions. 3. What is initiating structure? Consideration? Answer Initiating structure refers to the extent to which a leader is likely to define and structure his/ her role and those of employees in the search for goal attainment. It includes attempts to organize work, work relationships and goals. Consideration is described as the extent to which a person is likely to have job relationships that are characterized by mutual trust, respect for employees ideas, and regard for their feelings. Leaders high in initiating structure and consideration tended to achieve high employee performance and satisfaction. 4. What is the managerial grid? Contrast its approach to leadership with the approaches of the Ohio State and Michigan groups. Answer Blake and Mouton proposed a managerial grid based on the styles of concern for people and concern for production, which essentially represent the Ohio State dimensions of consideration and initiating structure or the Michigan dimensions of employee-oriented and production-oriented. The grid has nine possible positions along each axis, creating 81 different positions. See Exhibit 11-1. The grid shows the dominating factors in a leaders thinking in regard to getting results. Ohio State researchers sought to identify independent dimensions of leader behavior. They narrowed over a thousand dimensions into two categoriesinitiating structure and consideration. University of Michigan leadership studies were undertaken at the same time as those being done at Ohio State, with similar research objectives. They discovered two dimensions of leadership behavior employeeoriented and production-oriented. Employee-oriented leaders emphasized interpersonal relations; they took a personal interest in the needs of their employees and accepted individual differences among members. The production-oriented leaders tended to emphasize the technical or task aspects of the job; group members were a means to that end. 5. What was the contribution of the Scandinavian studies to the behavioral theories? Answer Researchers in Finland and Sweden have been reassessing the two-dimension model. Their basic premise is that effective leaders would exhibit development-oriented behavior. These leaders value experimentation, seek new ideas, and generate and implement change. The Scandinavian researchers review of the original Ohio State data found development items such as pushes new ways of doing things, originates new approaches to problems, and encourages members to start new activities. Yet these items did not explain much toward effective leadership. The Scandinavian researchers proposed that this was because developing new ideas and implementing change were not critical in those days. The Scandinavian researchers have been conducting new studies looking to see if there is a third dimensiondevelopment orientationthat is related to leader effectiveness. The early evidence is positive. Using samples of leaders in Finland and Sweden, the researchers have found strong support for development-oriented leader behavior as a separate and independent dimension. Previous behavioral approaches that focused in on only two behaviors may not appropriately capture leadership in the

1990s. 6. What are Fiedlers three contingency variables? Answer After assessing leadership style, it is necessary to match the leader with the situation. Fiedler identified three contingency variables to match the leader with the situation: Leader-member relationsThe degree of confidence, trust, and respect members have in their leader Task structureThe degree to which the job assignments are procedural. Position powerThe degree of influence a leader has over power variables such as hiring, firing, discipline, promotions, and salary increases. 7. What contribution does cognitive resource theory make to leadership? Answer Fiedler and Garcia reconceptualized the contingency model including the element of stress as a variable. The essence of the new theory is that stress is the enemy of rationality. It is difficult for leaders to think logically and analytically when they are under stress. The importance of a leaders intelligence and experience to his/her effectiveness differs under low- and high-stress situations. Intelligence and experience interfere with each other. Three conclusions: Directive behavior results in good performance only if linked with high intelligence in supportive, low-stress situations. In high stress situations, there is a positive relationship between job experience and performance. The intellectual abilities of leaders correlate with group performance in situations that the leader perceives as low in stress. 8. What are its implications for leadership practice? Answer The leader-member exchange (LMX) theory argues that because of time pressures, leaders establish a special relationship with a small group of their followers. These individuals make up the ingroup they are trusted, get a disproportionate amount of the leaders attention, and are more likely to receive special privileges. The theory proposes that early in the history of the interaction between a leader and a given follower, the leader implicitly categorizes the follower as an in or an out and that relationship is relatively stable over time. How the leader chooses who falls into each category is unclear. See Exhibit 11-3. The leader does the choosing on the basis of the followers characteristics. Research to test LMX theory has been generally supportive. The theory and research surrounding it provide substantive evidence that leaders do differentiate among followers and that these disparities are far from random. 9. What are the contingency variables in path-goal theory? Answer The contingency variables in path-goal theory are extracted from the key elements of initiating structure and consideration from the Ohio State studies and the expectancy theory of motivation. It says that the leaders job is to help followers in attaining their goals and to provide the necessary direction and support to ensure their goals are compatible with the overall objectives of the organization. 10. What are the implications if leaders are inflexible in adjusting their style? Answer Leaders will be ineffective. Directive leadership styles are redundant when employees have high ability and considerable experience. It is also ineffective when tasks are highly structured and well laid out. Employees in these situations prefer supportive leadership. Conversely, supportive leadership is less effective when tasks are ambiguous which would render a leader ineffective using that style.

QUESTIONS FOR CRITICAL THINKING 1. Review trait theories in the context of the nature vs. nurture debate. Answer Proponents of trait theories believe that a leader is born. They often describe leaders in terms of their personal characteristics, such as charismatic and driven. Behaviorist s believe leadership can be taught, or nurtured, by providing the necessary skills to an individual to be an effective leader. 2. If you were a manager, how would you assess a situation in terms of Fiedlers three contingency variables? Answer While each situation would no doubt require a different outcome, one should begin by analyzing the situation as to each variable. Are leader-member relations good or poor? If poor, work would need to be done to repair those relationships. For example, are these highly skilled people who have been subjected to a highly directive leadership style? It would be best to adjust that style to one that is more participative in nature. Second, task structure should be analyzed. Is there a great deal of flexibility in how the work is performed? Third, how much power does the leader have in hiring, firing and discipline? If the leader has little power, he or she will need to rely on a more participative style and use personal characteristics to influence others. Even with high position power, depending on the analysis of the other variables, a more participative leadership style would be in order. 3. Develop an example where you operationalize the Fiedler model. Answer Have students consider a class project, or team project situation where there is a need to balance getting something done (task) and develop social relations (relationship). Their operationalization ideas will vary but should include the following elements: Three contingency dimensions a. Leader-member relationsThe degree of confidence, trust, and respect members have in their leader b. Task structureThe degree to which the job assignments are procedural. c. Position powerThe degree of influence a leader has over power variables such as hiring, firing, discipline, promotions, and salary increases Three contingency variables a. Leader-member relations are either good or poor. b. Task structure is either high or low. c. Position power is either strong or weak. Matching leaders and situations (See Exhibit 11-2.) 4. Develop an example where you operationalize path-goal theory. Answer Students might consider a tutoring or coaching situation where one student is helping another set and achieve specific quantified goals. They would need to identify which of the four leadership behaviors is most effective. Directive leadership leads to greater satisfaction when tasks are ambiguous or stressful than when they are highly structured and well laid out. Supportive leadership results in high employee performance and satisfaction when employees are performing structured tasks. The participative leader consults with followers and uses their suggestions before making a decision. Employees with an internal locus of control will be more satisfied with a participative style. Achievement-oriented leadership will increase employees expectancies that effort will lead to high performance when tasks are ambiguously structured.

5. Develop an example where you operationalize SLT. Answer Since the best documented studies have isolated five such characteristicsthey have a vision, are willing to take risks to achieve that vision, are sensitive to both environmental constraints and follower needs, and exhibit behaviors that are out of the ordinarythe activities should provide an opportunity to develop and display these characteristics. See Exhibit 11-7. Most experts believe that individuals can be trained to exhibit charismatic behaviors. The text offers a threestep process. Activities that allow a person to develop these would help the student be more charismatic. First, an individual needs to develop the aura of charisma by maintaining an optimistic view. Second, an individual draws others in by creating a bond that inspires others to follow. Third, the individual brings out the potential in followers by tapping into their emotions. 1. Contrast the three types of trust. Relate them to your experience in personal relationships. Answer Students personal experiences will vary but their discussion should recognize the following elements. Deterrence-based trust The most fragileone violation or inconsistency can destroy the relationship. Based on fear of reprisal if the trust is violated. It will work only to the degree that punishment is possible, consequences are clear, and the punishment is actually imposed if the trust is violated. Knowledge-based trust Most organizational relationships are rooted in knowledge-based trust. Relies on information rather than deterrence. Knowledge of the other party and predictability of his or her behavior replaces the contracts, penalties, and legal arrangements more typical of deterrence-based trust. Predictability enhances trusteven if the other is predictably untrustworthy. This trust is not necessarily broken by inconsistent behavior, if you believe you can adequately explain or understand anothers apparent violation. Identification-based trust The highest level of trustachieved when there is an emotional connection between the parties. It allows one party to act as an agent for the other in interpersonal transactions. Trust exists because the parties understand each others intentions and appreciate the others wants and desires. Controls are minimal at this level. 2. What could you do if you wanted others to perceive you as a charismatic leader? Answer Since the best documented studies have isolated five such characteristics they have a vision, are willing to take risks to achieve that vision, are sensitive to both environmental constraints and follower needs, and exhibit behaviors that are out of the ordinarythe activities should provide an opportunity to develop and display these characteristics. Most experts believe that individuals can be trained to exhibit charismatic behaviors. The text offers a three-step process. Activities that allow a person to develop these would help the student be more charismatic. First, an individual needs to develop the aura of charisma by maintaining an optimistic view. Second, an individual draws others in by creating a bond that inspires others to follow. Third, the individual brings out the potential in followers by tapping into their emotions. 3. When can charisma be a liability? Answer Charisma appears to be most appropriate when the followers task has an ideological component or when the environment involves a high degree of stress and uncertainty. This may explain why, when charismatic leaders surface, it is more likely to be in politics, religion, wartime; or when a business firm is in its infancy or facing a life-threatening crisis. During times of non-crisis this type of leadership is unnecessary.

4. What are the qualities of a vision? Answer The key properties of a vision seem to be inspirational possibilities that are value centered, realizable, with superior imagery and articulation. Desirable visions fit the times and circumstances and reflect the uniqueness of the organization. Additionally, people in the organization must also believe that the vision is attainable and it should be perceived as challenging yet do-able. Visions that have clear articulation and powerful imagery are more easily grasped and accepted. 5. How does a leader increase self-leadership among his or her followers? Answer Proponents of self-leadership say that there are a set of processes through which individuals control their own behavior. The following ideas have been suggested: Model self-leadership. Practice self-observation, setting challenging personal goals, self-direction, and self-reinforcement. Then display these behaviors and encourage others to rehearse and then produce them. Encourage employees to create self-set goals. Having quantitative, specific goals is the most important part of self-leadership. Encourage the use of self-rewards to strengthen and increase desirable behaviors. In contrast, selfpunishment should be limited only to occasions when the employee has been dishonest or destructive.

Create positive thought patterns. Encourage employees to use mental imagery and self-talk to further
stimulate self-motivation. Create a climate of self-leadership. Redesign the work to increase the natural rewards of a job and focus on these naturally rewarding features of work to increase motivation. Encourage self-criticism. Encourage individuals to be critical of their own performance. 6. How does EI relate to leadership effectiveness? Answer IQ and technical skills are threshold capabilities. They are necessary, but not sufficient requirements for leadership. It is the possession of the five components of emotional intelligence selfawareness, self-management, self-motivation, empathy, and social skillsthat allows an individual to become a star performer. Without EI, a person can have outstanding training, a highly analytical mind, a long-term vision, and an endless supply of terrific ideas, but still not make a great leader. 7. How does one become an effective leader? Answer Leadership has increasingly become viewed as the management of meaning. They define organizational reality through the articulation of a vision. It is directed predominantly toward leaders at the top of organizations and has more relevance to explaining the success and failures of chief executives than of first-line supervisors. Framing is a way to use language to manage meaning. It is a way for leaders to influence how events are seen and understood. Framing is analogous to what a photographer does. When the photographer aims her camera and focuses on a specific shot, she frames her photo. Others then see what she wanted them to see. They see her point of view. 8. Why would a leader want to be a mentor? Answer The mentor-protg relationship gives the mentor unfiltered access to the attitudes and feelings of lower-ranking employees. It is a valuable communication channel that allows mentors to have news of problems before they become common knowledge to others in upper management. In addition, in terms of

leader self-interest, mentoring can provide personal satisfaction to senior executives. The opportunity to share knowledge with others can be personally rewarding for the mentor. 9. How is leadership an attribution? Answer The attribution framework accounts for the conditions under which people use leadership to explain organizational outcomes. When an organization has either extremely negative or extremely positive performance, people are prone to make leadership attributions to explain the performance. This tendency helps to account for the vulnerability of CEOs when their organizations suffer a major financial setback, and the credit for extremely positive financial resultsregardless of how much or how little they contributed. 10. Contrast substitutes and neutralizers for leadership. Answer Contrary to the arguments, leadership may not always be important. Data from numerous studies collectively demonstrate that, in many situations, whatever actions leaders exhibit are irrelevant. Certain individual, job, and organizational variables can act as substitutes for leadership or neutralize the leaders effect to influence his or her followers. Neutralizers make it impossible for leader behavior to make any difference to follower outcomes. They negate the leaders influence. Substitutes make a leaders influence not only impossible but also unnecessary. They act as a replacement. QUESTIONS FOR CRITICAL THINKING 1. What role do you think training plays in an individuals ability to trust others? For instance, does the training of lawyers, accountants, law enforcement personnel, and social workers take different approaches toward trusting others? Explain. Answer Students answers will vary on this. Certainly, the professionals named above all have specific code of conduct as prescribed by their profession requiring that they conduct their duties to a specified standard. This may affect how they trust others given they work with a variety of individuals with varying backgrounds, however, they are required to treat everyone with the same level of respect. In the case of the police officer, he/she may treat the traffic law offender with respect throughout the interaction, but not necessarily trust that individual not to do him or her harm. Their training requires that they approach the vehicle and conduct the interaction in a way that also maximizes his or her own safety.

2. How might an understanding of knowledge-based trust explain the reluctance of a person to change
jobs? Answer Most organizational relationships are rooted in knowledge-based trust. It exists when you have adequate information about someone to understand them well enough to be able to accurately predict their behavior. An individual may be reluctant to change jobs because of the unknown of the new organization or new supervisor. 3. Its not possible to be both a trusting boss and a politically astute leader. One requires op enness and the other requires concealment. Do you agree or disagree with this statement ? Explain. Answer Students answers will vary on this. The ideas are not necessarily mutually exclusive of each other. 4. As a new employee in an organization, why might you want to acquire a mentor? Why might women and minorities have more difficulty in finding a mentor than white males? Answer A mentor is a senior employee who sponsors and supports a less-experienced employee (a protg). The mentoring role includes coaching, counseling, and sponsorship. The advantages include

assistance as they develop skills, provide support and bolster self-confidence, and intervene on behalf of the protgs to lobby for visible assignments and rewards such as promotions and salary increases. Mentors are typically the ones who initiate the relationship, and most often choose someone they can relate to often younger versions of themselves. The majority of senior executives are white males (although progress has been made) making it difficult for minorities and females to be selected as protgs. 5. Is there an ethical problem if leaders focus more on looking like a leader than actually being one? Discuss. Answer Students answers vary on this. Possibly if the leader is misleading people, yet it has been argued that too much credit is given to leaders who have just been in the right place at the right time.

1. What are the disadvantages to conflict? What are its advantages? Answer Conflict can injure feelings, delay the work process, result in factions developing, etc. Conflict, however, can stimulate opinions, raise more and better ideas, air any problems or interpersonal conflicts so that they can be resolved, etc. 2. What is the difference between functional and dysfunctional conflict? What determines functionality? Answer Functionalconstructive forms of conflict support the goals of the group and improve its performance. Conflicts that hinder group performance are dysfunctional or destructive forms of conflict. Dysfunctional conflict depends on the type of conflict. Task conflict relates to the content and goals of the work. Low-to-moderate levels of task conflict are functional and consistently demonstrate a positive effect on group performance because it stimulates discussion of ideas that help groups perform better. Relationship conflict focuses on interpersonal relationships. These conflicts are almost always dysfunctional. The friction and interpersonal hostilities inherent in relationship conflicts i ncrease personality clashes and decrease mutual understanding, which hinders the completion of organizational tasks. Process conflict relates to how the work gets done. Low-levels of process conflict are functional and could enhance team performance. For process conflict to be productive, it must be kept low. Intense arguments create uncertainty. 3. Under what conditions might conflict be beneficial to a group? Answer The conditions differ according to the type of conflict. With task conflict, low-to-moderate levels of task conflict are functional and stimulate discussion of ideas that help groups perform better. Relationship conflict is almost always dysfunctional because it decreases mutual understanding, which hinders the completion of organizational tasks. Process conflict is functional if kept to a low-level. 4. What are the components in the conflict process model? From your own experiences, give an example of how a conflict proceeded through the five stages. Answer The process is diagrammed in Exhibit 14-1. Stage I: Potential opposition or incompatibilityThe first step in the conflict process is the presence of conditions that create opportunities for conflict to arise. These conditions have been condensed into three general categories: communication, structure, and personal variables. Stage II: Cognition and personalizationThe antecedent conditions can lead to conflict only when one or more of the parties are affected by, and aware of, the conflict. Just because a conflict is perceived does

not mean that it is personalized. It is important because it is where conflict issues tend to be defined. Stage III: IntentionsIntentions are decisions to act in a given way. Exhibit 14-2 represents one authors effort to identify the primary conflict-handling intentions. Two dimensions cooperativeness and assertiveness. Five conflict-handling intentions can be identified: competing (assertive and uncooperative), collaborating (assertive and cooperative), avoiding (unassertive and uncooperative), accommodating (unassertive and cooperative), and compromising (midrange on both assertiveness and cooperativeness). Stage IV: BehaviorThe behavior stage includes the statements, actions, and reactions made by the conflicting parties. These conflict behaviors are usually overt attempts to implement each partys intentions. Exhibit 14-3 provides a way of visualizing conflict behavior. Exhibit 14-4 lists the major resolution and stimulation techniques that allow managers to control conflict levels. Stage V: OutcomesOutcomes may be functional in that the conflict results in an improvement in the groups performance, or dysfunctional in that it hinders group performance. Conflict is constructive when it improves the quality of decisions, stimulates creativity and innovation, etc. Dysfunctional outcomes uncontrolled opposition breeds discontent, which acts to dissolve common ties, and eventually leads to the destruction of the group. Among the more undesirable consequences are a retarding of communication, reductions in group cohesiveness, and subordination of group goals to the primacy of infighting between members. 5. How could a manager stimulate conflict in his or her department? Answer If managers accept the inter-actionist view toward conflict, they encourage functional conflict. Creating functional conflict is a tough job, particularly in large American corporations. A high proportion of people who get to the top are conflict avoiders. At least seven out of ten people in American business hush up when their opinions are at odds with those of their superiors, allowing bosses to make mistakes even when they know better. Examples of creating functional conflict: Hewlett-Packard rewards dissenters by recognizing go-against-the-grain types. Herman Miller, Inc., has a formal system in which employees evaluate and criticize their bosses. IBM also has a formal system that encourages dissension. Employees can question their boss with impunity. Royal Dutch Shell Group, General Electric, and Anheuser -Busch build devils advocates into the decision process. 6. What defines the settlement range in distributive bargaining? Answer The essence of distributive bargaining is depicted in Exhibit 14-6. Each partys target point or resistance point marks the lowest outcome that is acceptable, and the area between these two points makes up each partys aspiration range. As long as there is some overlap between A and Bs aspiration ranges, there exists a settlement range where each ones aspirations can be met. 7. Why isnt integrative bargaining more widely practiced in organizations? Answer In terms of intra-organizational behavior, all things being equal, integrative bargaining is preferable to distributive bargaining, because the former builds long-term relationships and facilitates working together in the future. It bonds negotiators and allows each to leave the bargaining table feeling that he or she has achieved a victory. We do not see more integrative bargaining in organizations because certain conditions are necessary for this type of negotiation to succeed. Parties who are open with information and candid about their concerns A sensitivity by both parties to the others needs The ability to trust one another A willingness by both parties to maintain flexibility

8. How do men and women differ, if at all, in their approaches to negotiation? Answer Men and women do not negotiate differently. Comparisons between experienced male and female managers find women are neither worse nor better negotiators, neither more cooperative nor open to the other, and neither more nor less persuasive nor threatening than are men. However, womens attitudes toward negotiation and toward themselves as negotiators appear to be quite different from mens. Managerial women demonstrate less confidence in anticipation of negotiating and are less satisfied with their performance despite achieving similar outcomes as men. Women may unduly penalize themselves by failing to engage in negotiations when such action would be in their best interests. 9. What problems might Americans have in negotiating with people from collectivist cultures like China and Japan? Answer The Chinese also draw out negotiations but that is because they believe negotiations never end. Just when you think you have reached a final solution, the Chinese executive might smile and start the process all over again. Like the Japanese, the Chinese negotiate to develop a relationship and a commitment to work together. Americans are known around the world for their impatience and their desire to be liked. Astute negotiators often turn these characteristics to their advantage. North Americans tried to persuade by relying on facts and appealing to logic. They made small concessions early in the negotiation to establish a relationship, and usually reciprocated opponents concessions. North Americans treated deadlines as very important. Another study looked at verbal and nonverbal negotiation tactics exhibited by North Americans and Japanese. Japanese on average said No five times for the nine times the North Americans did. The Japanese displayed more than five periods of silence lasting longer than ten seconds during the 30-minute sessions. North Americans averaged 3.5 such periods. The Japanese and North Americans interrupted their opponent about the same number of times. Finally, the Japanese and the North Americans had no physical contact with their opponents during negotiations except for handshaking. 10. What can you do to improve your negotiating effectiveness? Answer Take the time to assess your own goals, consider the other partys goals and interests, and develop a strategy, then try the following: Begin with a positive overture. Concessions tend to be reciprocated and lead to agreements. Address problems, not personalities. Concentrate on the negotiation issues, not on the personal characteristics of your opponent. Pay little attention to initial offers. Treat an initial offer as merely a point of departure. Emphasize win-win solutions, assuming a zero-sum game means missed opportunities for trade-offs that could benefit both sides. So, if conditions are supportive, look for an integrative solution. Create an open and trusting climate. Skilled negotiators are better listeners , ask more questions, focus their arguments more directly, are less defensive, and have learned to avoid words and phrases that can irritate an opponent.

QUESTIONS FOR CRITICAL THINKING 1. Do you think competition and conflict are different? Explain. Answer They are both different and the same. First, they are the same in that there is a struggle over an issue, a resource, a decision between two or more parties, and a fight for control or power whether its winning a race or arguing over how to handle a labor dispute. They are the same in that they can be personal or professional in nature, but they are also different. Supposedly, competition is to stay on the field of competition, and not become personal, whereas a major source of conflict is person al. Competition is universally valued in American culture, whereas its still a split decision over the benefits of conflict. 2. Participation is an excellent method for identifying differences and resolving conflicts. Do you agree or disagree? Discuss. Answer Participation will do this if there is trust, training in how to facilitate non-personal disagreement, and a commitment to work together. If these are not present, then participation is worthless. 3. From your own experience, describe a situation you were involved in where the conflict was dysfunctional. Describe another example, from your experience, where the conflict was functional. Now analyze how other parties in both conflicts might have interpreted the situation in terms of whether the conflicts were functional or dysfunctional. Answer Students examples will vary but should take the criteria for functional and dysfunctional conflict into consideration; see the answer for #2 above in Questions for Review. 4. Assume a Canadian had to negotiate a contract with someone from Spain. What problems might he or she face? What suggestions would you make to help facilitate a settlement? Answer The text does not provide information directly related to these two nationalities. Students might draw the following assumptions from two parallel cultures Brazilians and North Americansor you may wish to assign a brief cultural background research assignment to students. Consider bringing in a colleague from the Modern Languages department to discuss Spanish culture. 5. Michael Eisner, CEO at the Walt Disney Co., wants to stimulate conflict inside his firm, but he wants to minimize conflict with outside partiesagents, contractors, unions, etc. What does this say about conflict levels, functional vs. dysfunctional conflict, and managing conflict? Answer It suggests that there may be apathy or groupthink going on and Mr. Eisner wants to create more energy inside the firm. He is probably looking for new ideas, increased communication, etc. He does not, however, want to create negative relationships with outside parties. He is looking for functional conflict to improve performance and is not afraid of the challenge to do so. 1. Why isnt work specialization an unending source of increased productivity? Answer The essence of work specialization is that an entire job is broken down into a number of steps, each step being completed by a separate individual. By the late 1940s, most manufacturing jobs in industrialized countries were being done this way. For much of the first half of this century, managers viewed work specialization as an unending source of increased productivity. By the 1960s, there became increasing evidence that a good thing can be carried too far. The point had been reached in some jobs where the human

diseconomies from specializationwhich surfaced as boredom, fatigue, stress, low productivity, poor quality, increased absenteeism, and high turnovermore than offset the economic advantages. See Exhibit 15-2. 2. All things being equal, which is more efficienta wide or narrow span of control? Why? Answer How many employees a manager can efficiently and effectively direct depends on a number of factors. Wider spans are more efficient in terms of cost. However, at some point, wider spans reduce effectiveness. That is, when the span becomes too large, employee performance suffers because supervisors no longer have the time to provide the necessary leadership and support. Narrow or small spans have their advocates. By keeping the span of control to five or six employees, a manager can maintain close control, but narrow spans have three major drawbacks. First, as already described, they are expensive because they add levels of management. Second, they make vertical communication in the organization more complex. The added levels of hierarchy slow down decision making and tend to isolate upper management. Third, narrow spans of control encourage overly tight supervision and discourage employee autonomy. The trend in recent years has been toward wider spans of control. Managers recognize that they can handle a wider span when employees know their jobs inside and out or can turn to their coworkers when they have questions. All things being equal, the wider or larger the span, the more efficient the organization. 3. In what ways can management departmentalize? Answer Grouping jobs together so common tasks can be coordinated is called departmentalization. One of the most popular ways to group activities is by functions performed. Tasks can also be departmentalized by the type of product the organization produces. Another way to departmentalize is on the basis of geography or territory. Process departmentalization is exemplified by Reynolds Metals aluminum tubing plant where production is organized into five departments: casting, press, tubing, finishing, and inspecting, packing, and shipping. A final category of departmentalization is by type of customer. Large organizations may use all of the forms of departmentalization that we have described. 4. What is a matrix structure? When would management use it? Answer The matrix structure is used in advertising agencies, aerospace firms, research and development laboratories, construction companies, hospitals, government agencies, universities, management consulting firms, and entertainment companies. It combines two forms of departmentalization: functional and product. The matrix attempts to gain the strengths of both functional and product departmentalization, while avoiding their weaknesses. The most obvious structural characteristic of the matrix is that it breaks the unityofcommand concept. Employees in the matrix have two bosses their functional department managers and their product managers. Therefore, the matrix has a dual chain of command. The strength of the matrix lies in its ability to facilitate coordination when the organization has a multiplicity of complex and interdependent activities. It facilitates the efficient allocation of specialists. The matrix achieves the advantages of economies of scale by providing the organization with both the best resources and an effective way of ensuring their efficient deployment. The major disadvantages of the matrix lie in the confusion it creates, its propensity to foster power struggles, and the stress it places on individuals. As an organization gets larger, its information processing capacity can become overloaded. In a bureaucracy, complexity results in increased formalization. The direct and frequent contact between different

specialties in the matrix can make for better communication and more flexibility. Information permeates the organization and more quickly reaches those people who need to take account of it. Furthermore, the matrix reduces bureaupathologies. The dual lines of authority reduce tendencies of departmental members to become so busy protecting their little worlds that the organizations overall goals become secondary. 5. Contrast the virtual organization with the boundaryless organization. Answer The Virtual Organization: The essence of the virtual organization is typically a small, core organization that outsources major business functions. It is highly centralized, with little or no departmentalization. Virtual organizations create networks of relationships that allow them to contract out manufacturing, distribution, marketing, or any other business function where management feels that others can do it better or more cheaply. The virtual organization stands in sharp contrast to the typical bureaucracy in that it outsources many generic functions and concentrates on what it does best. The major advantage to the virtual organization is its flexibility. The primary drawback is that it reduces managements control over key parts of its business. The Boundaryless Organization: The boundaryless organization seeks to eliminate the chain of command, have limitless spans of control, and replace departments with empowered teams. Because it relies so heavily on information technology, some call this structure the T-form (or technology-based) organization. By removing vertical boundaries, management flattens the hierarchy and: Minimizes status and rank. Uses cross-hierarchical teams. Uses participative decision-making practices. Uses 360-degree performance appraisals. Functional departments create horizontal boundaries. The way to reduce these barriers is to replace functional departments with cross-functional teams and to organize activities around processes. When fully operational, the boundaryless organization also breaks down barriers to external constituencies (suppliers, customers, regulators, etc.) and barriers created by geography. The one common technological thread that makes the boundaryless organization possible is networked computers. 6. What type of structure works best with an innovation strategy? A cost-minimization strategy? An imitation strategy? Answer An organizations structure is a means to help management achieve its objectives. Structure should follow strategy. An innovation strategy means a strategy for meaningful and unique innovations, such as 3M Company. A cost-minimization strategy tightly controls costs, refrains from incurring unnecessary innovation or marketing expenses, and cuts prices in selling a basic product, such as Wal-Mart. An imitation strategy tries to capitalize on the best of bothminimize risk and maximize opportunity for profit, move into new products or new markets only after viability has been proven by innovators, and copy successful ideas of innovators. Innovators need the flexibility of the organic structure. Cost minimizers seek the efficiency and stability of the mechanistic structure. Imitators combine the two structures.

A mechanistic structure in order to maintain tight controls and low costs Organic subunits in which to pursue new undertakings 7. Summarize the size-structure relationship. Answer There is considerable evidence to support that an organizations size significantly affects its structure. However, size affects structure at a decreasing rate. The impact of size becomes less important as an organization expands. Once an organization has around 2,000 employees, it is already fairly mechanistic. An additional 500 employees will not have much impact. On the other hand, adding 500 employees to an organization that has only 300 members is likely to result in a shift toward a more mechanistic structure. 8. Define and give an example of what is meant by the term "technology." Answer Technology refers to "how an organization transfers its inputs into outputs." Every organization has at least one technology for converting financial, human, and physical resources into products or services. The Ford Motor Co., for instance, predominantly uses an assembly-line process to make its products. On the other hand, colleges may use a number of instruction technologies the ever-popular formal lecture method, the case analysis method, the experiential exercise method, the programmed learning method, and so forth. 9. Summarize the environment-structure relationship. Answer An organizations environment is composed of those institutions or forces that are outside the organization and potentially affect the organizations performance. Environmental uncertainty affects structure. Static environments are those in which few forces in their environment are changing. Dynamic environments are those that are rapidly changing. Static environments create significantly less uncertainty than do dynamic ones. Organizational structure is one way to reduce environmental uncertainty. There are three key dimensions to any organizations environmentcapacity, volatility, and complexity. Exhibit 1510 summarizes our definition of the environment along its three dimensions. Some general conclusions: There is evidence that relates the degrees of environmental uncertainty to different structural arrangements. Specifically, the more scarce, dynamic, and complex the environment, the more organic a structure should be. The more abundant, stable, and simple the environment, the more the mechanistic structure will be preferred. 10. Explain the importance of the statement: Employees form implicit models of organizational structure. Answer One cannot generalize when linking organizational structures to employee performance and satisfaction. There is no predominant preference among employees. Generally, work specialization contributes to higher employee productivity but reduces job satisfaction. There seems to be no evidence to support a relationship between span of control and employee performance. There is a fairly strong link between centralization and job satisfaction. To maximize employee performance and satisfaction, individual differences, such as experience, personality, and the work task, should be taken into account. In addition, national culture influences preference for structure so it, too, needs to considered. There is substantial evidence that individuals are attracted to, selected by, and stay with organizations that suit their personal characteristics. The effect of structure on employee behavior is undoubtedly reduced where the selection process facilitates proper matching of individual characteristics with organizational characteristics.

QUESTIONS FOR CRITICAL THINKING 1. How is the typical large corporation of today organized in contrast to how that same organization was probably organized in the 1960s? Answer Today corporations are flatter, less layers of management, more lower-level workers performing managerial functions, more use of technology for accessing, mana ging, and using information to make decisions. Also, there is more use of teams, especially cross-functional teams. 2. Do you think most employees prefer high formalization? Support your position. Answer Formalization refers to the degree to which jobs within the organization are standardized. If a job is highly formalized, then the job incumbent has a minimum amount of discretion over what is to be done, when it is to be done, and how he or she should do it. Where formalization is low, job behaviors are relatively nonprogrammed and employees have a great deal of freedom to exercise discretion in their work. Since an individuals discretion on the job is inversely related to the amount of behavior in that job that is preprogrammed by the organization, the greater the standardization, the less input the employee has into how his/her work is to be done. The degree of formalization can vary widely between organizations and within organizations. The argument for employee preferences needs to take into concern types of jobs, employee goals and ambitions, and the payoff for less formalization. Some people seek jobs with minimal responsibility for their own reasons. Also, if the rewards for accepting more responsibility are not adequate, employees will prefer higher formalization. 3. If you were an employee in a matrix structure, what pluses do you think the structure would provide? What about minuses? Answer Plusesvariety in work, the opportunity to get to know the entire business, more opportunities without having to be promoted. Minusesconfusion of who is in charge, difficulty in setting priorities, constant tension over who you are in the sense of your functional area, etc. 4. What behavioral predictions would you make about people who worked in a pure boundaryless organization (if such a structure were ever to exist)? Answer The boundaryless organization seeks to eliminate the chain of command, have limitless spans of control, and replace departments with empowered teams. Because it relies so heavily on information technology, some call this structure the T-form (or technology-based) organization. One might expect a more egalitarian climate, greater initiative by rank and file employees, better communication and listening both up and down the management hierarchy that remains. Also, one may expect cross-functional teams to organize activities around processes. There are also fewer barriers to external constituencies. 5. AOL buys Time Warner. Alcoa purchases Reynolds Metals. Nestles S.A. merges with Ralston Purina. Each of these is a recent example of large companies combining with other large companies. Does this imply that small is not necessarily beautiful? Are mechanistic forms winning the survival of the fittest battle? What are the implications of this consolidation trend for organizational behavior? Answer Some bureaucratic characteristics are in decline but have not entirely disappeared from todays organizations. It is hard to draw any implications because some factors affecting merger decisions relate to image, stock price, and resources, none of which are inherently tied to effectiveness in organizational

functioning. Students comments should take into consideration the following items. Organizations that succeed and survive tend to grow to large size, and bureaucracy is efficient with large size. The impact of uncertainties in the environment on the organization are substantially reduced by management strategies such as environmental scanning, strategic alliances, advertising, and lobbying. Bureaucracys goal of standardization can be increasingly achieved through hiring people who have undergone extensive educational training. Technology has replaced some previously bureaucratic characteristics, but without any loss of management control.

1. What are the implications for employees of a continuous improvement program?


Answer Continuous improvement programs like total quality management (TQM) represent a philosophy of management thats driven by the constant attainment of customer satisfaction through the continuous improvement of all organizational processes. TQM programs seek to achieve continuous process improvements reducing variability. Increasing the uniformity of the product or service results in lower costs and higher quality. What this means for employees and their jobs is: no ability to rest on their previous accomplishments and successes. increased stress from a work climate that no longer accepts complacency with the status quo. a race with no finish line means a race thats never over, which creates constant tension. the pressures from an unrelenting search for process improvements can create anxiety and stress. probably the most significant implication for employees is that management will look to them as the prime source for improvement ideas.

2. What are the implications for employees of a process reengineering program?


Answer Many people have lost, and will continue to lose, their jobs due to reengineering efforts. Staff support jobs, especially middle managers, will be most vulnerable. So, too, clerical jobs in service industries. Those employees that keep their jobs will find that the jobs arent the same. These new jobs will typically require a wider range of skills, include more interaction with customers and suppliers, offer greater challenge, contain increased responsibilities, and provide higher pay. The 3- to 5-year period it takes to implement reengineering is usually tough on employees.

3. Contrast e-commerce with and e-organization


Answer -- E-commerce refers to the sales side of electronic business. For example, people shopping in the Internet, businesses setting up web sites, fulfill orders and get paid are all parts of e-commerce. Its a subset of e-business. E-organizations refers to the applications of e-business concepts to all organizations. The e-organization has three underlying components: The Internet a world wide network of interconnected computers, Intranetsan organizations private Internet, and Extranetsextended Intranets accessible only to selected employees and authorized outsiders

4. How can managers reduce cyberloafing?


Answer -- Cyberloafing refers to using the organizations Internet access during formal work hours to surf non-job related Web sites or sending/receiving personal email. If work isnt interesting workers may be motivated to do something elseoften surfing the Internet is the diversion. Solutions are to make jobs more interesting, provide formal breaks, and set out explicit guidelines for behavior.

5. What are the implications of the social information processing model for predicting employee behavior?
Answer People can look at the same job and evaluate it differently. The fact that people respond to their jobs as they perceive them rather than to the objective jobs themselves is the central thesis of the social information processing (SIP) model. The SIP model argues that employees adopt attitudes and behaviors in response to the social cues provided by others with whom they have contact. A number of studies generally confirm the validity of the SIP model. They show that employee motivation and satisfaction can be manipulated by such subtle actions as a coworker or boss commenting on the existence or absence of job features like difficulty, challenge, and autonomy. So managers should give as much (or more) attention to employees perceptions of their jobs as to the actual characteristics of those jobs. They might spend more time telling employees how interesting and important their jobs are. And managers should also not be surprised that newly hired employees and people transferred or promoted to a new position are more likely to be receptive to social information than are those with greater seniority.

6. How could you design an office so as to increase the opportunity for employees to be productive?
Answer Students responses will vary based on their experience with and desire for interaction and privacy when working. There is no right answer but students should consider the various elements that effect office design and be able to defend their design using those principles.

7. What is feng shui?


Answer -- An ancient Chinese system for arranging a persons surroundings so they are in harmony and balance with nature. The single most important concept is chi or life force. Managers should layout buildings and offices so as to utilize chi.

8. What are the advantages of flextime from an employees perspective? From managements
perspective? Answer Flextime is a scheduling option that allows employees, within specific parameters, to decide when to go to work. Flexible work hours allow employees some discretion over when they arrive at and leave work. The management benefits include reduced absenteeism, increased productivity, reduced overtime expenses, a lessening in hostility toward management, reduced traffic congestion around work sites, elimination of tardiness, and increased autonomy and responsibility for employees which may increase employee job satisfaction. For employees, more control over their work schedule, the ability to miss rush hour traffic, the ability to schedule work for their most productive time of day, etc. 9. What are the advantages of job sharing from an employees perspective? From managements perspective? Answer For employees, it allows two or more individuals to split a traditional 40-hour-a-week job, and it increases flexibility. For companies, job sharing allows the firm to draw upon the talents of more than one individual in a given job. It creates the opportunity to acquire skilled workers who might not be available on a full-time basis.

10. From an employees perspective, what are the pros and cons of telecommuting?
Answer Telecommuting refers to employees who do their work at home at least two days a week on a computer that is linked to their office. Three categories of jobs lend themselves to telecommuting: routine information-handling tasks, mobile activities, and professional and other knowledge-related tasks. Telecommuting increases employee flexibility, improves employee morale, etc. QUESTIONS FOR CRITICAL THINKING 1. Process reengineering needs to be autocratically imposed in order to overcome employee resistance. This runs directly counter to the model of a contemporary manager who is a good listener, a coach, motivates through employee involvement, and who possesses strong team support skills. Can these two positions be reconciled? Answer The managers job is to lead and may need to resort to a more autocratic approach in the beginning when employees are resistant to change. However, as process reengineering begins to take hold, employees jobs are often more interesting than before and they prefer the new work challenges. Often they are now a part of flexible teams and more autonomousallowing the manager to be more coach like as opposed to directive.

2. How has technology changed the managers job over the past 20 years?
Answer It allows the manager to manage/lead more people. It permits long distance work relationships. There are fewer clerical workers, and the manager does more of his/her own clerical work, typing, etc.

3. What effect, if any, do you think the Internet will have on the development of future employees
interpersonal skills? Answer The Internet allows employees more flexibility and accessibility than ever before. Employees will need to continue to develop communication and teamwork skills, but expand the traditional thinking and incorporate these skills with people they may never meet face-to-face while working solely over a computer screen.

4. Describe three jobs that score high on the JCM. Describe three jobs that would score low. Explain how
you came to your conclusions? Answer See Exhibit 16-1 for examples of job activities that rate high and low for each characteristic. Also, Exhibit 16-2 presents the model.

5. What can management do to improve employees perceptions that their jobs are interesting and
challenging? Answer Management can emphasize what the employees have in the way of positive job characteristics. They can explain to employees how their jobs fit into the overall mission of the organization. They can use titles and other symbols of status to elevate the image of certain jobs, etc.

1. What is job analysis? How is it related to those the organization hires? Answer Job analysis involves developing a detailed description of the tasks involved in a job, determining

the relationship of a given job to other jobs, and ascertaining the knowledge, skills, and abilities necessary for an employee to successfully perform the job. Exhibit 17-1 describes the more popular job analysis methods. 2. What are assessment centers? Why do you think they might be more effective for selecting managers than traditional written tests? Answer Assessment centers are a more elaborate set of performance simulation tests, specifically designed to evaluate a candidates managerial potential. Line executives, supervisors, and/or trained psychologists evaluate candidates as they go through one to several days of exercises that simulate real problems that they would confront on the job. Based on a list of descriptive dimensions that the actual job incumbent has to meet, activities might include interviews, in-basket problem-solving exercises, leaderless group discussions, and business decision games. For instance, a candidate might be required to play the role of a manager who must decide how to respond to ten memos in his/her in-basket within a two-hour period. Assessment centers have consistently demonstrated results that predict later job performance in managerial positions. 3. Contrast formal and informal training. Answer Historically, training meant formal training. It is planned in advance and has a structured format. Organizations are increasingly relying on informal trainingunstructured, unplanned, and easily adapted to situations and individuals. Most informal training is nothing other than employees helping each other out. They share information and solve work-related problems with one another. 4. What can organizations do to help employees develop their careers? Answer Amoco Corporations career development program is a model for modern companies. It is designed around employee self-reliance and to help employees reflect on their marketability both inside and outside the firm. The essence of a progressive career development program is built on providing support for employees to continually add to their skills, abilities, and knowledge. This support includes: Clearly communicating the organizations goals and future strategies Creating growth opportunities Offering financial assistance Providing the time for employees to learn 5. What can individuals do to foster their own career development? Answer Employees should manage their own careers like entrepreneurs managing a small business. The following suggestions are consistent with the view that you, and only you, hold primary responsibility for your career: Know yourself. Manage your reputation. Build and maintain network contacts. Keep current. Balance your specialist and generalist competencies. Document your achievements. Keep your options open.

6. Why do organizations evaluate employees? Answer See Exhibit 17-3 for survey results on primary uses of evaluations. Management uses evaluations for general human resource decisions. Decisions such as promotions, transfers, and terminations; Evaluations identify training and development needs. They pinpoint employee skills and competencies needing development. Criterion against which selection and development programs are validated The effectiveness of training and development programs can be assessed by examining the subsequent performance evaluations of participants. Providing feedback to employees on how the organization views their performance 7. What are the advantages and disadvantages of the following performance evaluation methods: (a) written essays, (b) graphic rating scales, and (c) behaviorally anchored rating scales? Answer Written essays The simplest method of evaluation is to write a narrative describing an employees strengths, weaknesses, past performance, potential, and suggestions for improvement. No complex forms or extensive training is required, but the results often reflect the ability of the writer. Graphic ratings scales These are one of the oldest and most popular methods of evaluation. A set of performance factors, such as quantity and quality of work, depth of knowledge, cooperation, loyalty, attendance, honesty, and initiative, is listed. The evaluator then goes down the list and rates each on incremental scales. The scales typically specify five points. These scales are popular because they are less time-consuming to develop and administer and allow for quantitative analysis and comparison, but they do not provide the depth of information that essays or critical incidents do. Behaviorally anchored rating scales BARS combine major elements from the critical incident and graphic rating scale approaches. The appraiser rates the employees based on items along a continuum, but the points are examples of actual behavior. BARS specify definite, observable, and measurable job behavior. Examples of job-related behavior and performance dimensions are found by asking participants to give specific illustrations of effective and ineffective behavior regarding each performance dimension. The results of this process are behavioral descriptions, such as anticipates, plans, executes, solves immediate problems, carries out orders, and handles emergency situations. 8. How can management effectively evaluate individuals when they work as part of a team? Answer Performance evaluation concepts have been almost exclusively developed with only individual employees in mind. Four suggestions for designing a system that supports and improves the performance of teams are: Tie the teams results to the organizations goals. Begin with the teams customers and the work process the team follows to satisfy customers needs. Measure both team and individual performance. Train the team to create its own measures. 9. How can an organizations performance evaluation system affect employee behavior? Answer A vital component of expectancy model of motivation is performance, specifically the effortperformance and performance-reward linkages. Performance is defined by the individuals performance evaluation. To maximize motivation, people need to perceive that the effort they exert leads to a favorable

performance evaluation and that the favorable evaluation will lead to the rewards that they value. Individuals will work considerably below their potential: If objectives are unclear. if criteria for measuring those objectives are vague. if they lack confidence in their efforts. 10. What impact do unions have on an organizations reward system? Answer For employees who are members of a labor union, wage levels and conditions of employment are explicitly articulated in a contract that is negotiated, through collective bargaining, between representatives of the union and the organizations management. The most obvious and pervasive area of labors influence is wage rates and working conditions. Where unions exist, performance evaluation systems tend to be less complex. The union contract affects motivation through determination of wage rates, seniority rules, layoff procedures, promotion criteria, and security provisions. QUESTIONS FOR CRITICAL THINKING 1. How could the phrase the best predictor of future behavior is past behavior guide you in managing human resources? Answer Since the best predictor of future behavior is past behavior, the best interview questions tend to be those that focus on previous experiences that are relevant to the current job. Examples might include: What have you done in previous jobs that demonstrates your creativity? or On your last job, what was it that you most wanted to accomplish but didnt? Why didnt you? 2. Describe a training program you might design to help employees develop their interpersonal skills. How would that program differ from one you designed to improve employee ethical behavior? Answer You may need to help lead students through this answer as the text does not address training design issues. Students responses should address the following facts and categories of interpersonal skills: Almost all employees belong to a work unit. To some degree, their work performance depends on their ability to effectively interact. These skills include how to be a better listener, how to communicate ideas more clearly, and how to be a more effective team player. Their responses should also consider the fact that individuals process, internalize, and remember new and difficult material differently, and therefore strategies should be individualized to the learning style of the specific employee. 3. What relationship, if any, is there between job analysis and performance evaluation? Answer Job analysis provides the defensible basis for performance evaluation based on task outcomes or behavior. Job analysis ensures that the performance evaluation evaluates job relevant performance. 4. What problems, if any, can you see developing as a result of using 360-degree evaluations? Answer 360-degree evaluations are the latest approach to performance evaluation. It provides for performance feedback from the full circle of daily contacts that an employee might have. See Exhibit 174.

The administration could be cumbersome as more people know about the individuals evaluation. Because they contributed, individuals might be reluctant to make tough decisions that will anger peers or subordinates for fear of reprisals later in their evaluations, etc. 5. GE prides itself on continually raising the performance bar by annually letting go employees who perform in the lowest 10 percent. In contrast, Cleveland-based Lincoln Electric Co. prides itself on its no-layoff policy. Lincoln Electric has provided its employees with guaranteed-employment since 1958. How can two successful companies have such different approaches to employment security? How can they both work? What implications can you derive from the success of these different practices? Answer Students will have a variety of responses to this question. Typically the responses would include a reference to organizational goals and value systems or culture in terms of making this type of system work. Also, selection practices will also be a consideration. Not everyone will want to work in a rate and rank system, yet others will find it extremely challenging and motivating. The implications of each system include how each organization spends its resources in terms of training, retaining, market conditions, etc. 1. Whats the difference between job satisfaction and organizational culture? Answer Organizational culture is concerned with how employees perceive the characteristics of an organizations culture, not with whether or not they like them. That is, it is a descriptive term. This is important because it differentiates this concept from that of job satisfaction. Research on organizational culture has sought to measure how employees see their organization: Does it encourage teamwork? Does it reward innovation? Does it stifle initiative? In contrast, job satisfaction seeks to measure affective responses to the work environment. It is concerned with how employees feel about the organizations expectations, reward practices, and the like. Although the two terms undoubtedly have overlapping characteristics, keep in mind that the term organizational culture is descriptive, while job satisfaction is evaluative. 2. Can an employee survive in an organization if he/she rejects its core values? Explain. Answer Organizational culture is a system of shared meaning held by members that distinguishes the organization from other organizations. This system of shared meaning is a set of key characteristics that the organization values. Organizational culture is concerned with how employees perceive its characteristics, not if they like them. Job satisfaction seeks to measure affective responses to the work environmenthow employees feel about the organizations expectations, reward practices, and the like. Therefore, if an individual does not fit the organizations culture, the individual will have some significant difficulty in surviving, let alone growing. The dominant culture expresses the core values that are shared by a majority. Not sharing those values places the employee on the outside socially and organizationally. 3. What defines an organizations subcultures? Answer Subcultures tend to develop in large organizations to reflect common problems, situations, or experiences that members face. They are defined by department designations and geographical separation. It will include the core values plus additional values unique to members of the subculture. The core values are

essentially retained but modified to reflect the subculture. Many organizations also have subcultures that can influence the behavior of members. 4. Contrast organizational culture with national culture. Answer The opening example in the text showed how Japans national culture was closely intertwined with corporate culture. National cultures must be taken into account if accurate predictions are to be made about organizational behavior in different countries. The research indicates that national culture has a greater impact on employees than does their organizations culture. This has to be qualified to reflect the selfselection that goes on at the hiring stage. The employee selection process will be used by multinationals to find and hire job applicants who are a good fit with their organizations dominant culture. National culture is ones primary culture generally has deep roots. Organizational culture is something one subscribes to and may adapt to, and even be somewhat affected by, but only to the degree it fits ones own national culture. 5. How can culture be a liability to an organization? Answer While organizational culture enhances organizational commitment and increases the consistency of employee behavior, there are potentially dysfunctional aspects of culture. It can be a barrier to change when the shared values are not in agreement with those that will further the organizations effectiveness. This is most likely to occur when an organizations environment is dynamic. It is inherently a barrier to diversity because hiring new employees who (because of race, gender, disability, or other differences) are not like the majority of the organizations members creates a paradox. Management wants new employees to accept the organizations core cultural values but, at the same time, wants to support the differences that these employees bring to the workplace. Strong cultures, therefore, can be liabilities when they effectively eliminate the unique strengths that diverse people bring to the organization or if they support institutional bias or become insensitive to people who are different. Finally, strong cultures can be a barrier to acquisitions and mergers. Historically, the key factors that management looked at in making acquisition/merger decisions were financial advantages and product synergy. Cultural compatibility has become the primary concern. Whether the acquisition actually works seems to have more to do with how well the two organizations cultures match up. 6. How does a strong culture affect an organizations efforts to improve diversity? Answer It is inherently a barrier to diversity because hiring new employees who (because of race, gender, disability, or other differences) are not like the majority of the organizations members creates a paradox. Management wants new employees to accept the organizations core cultural values but, at the same time, they want to support the differences that these employees bring to the workplace. Strong cultures put considerable pressure on employees to conform. They limit the range of values and styles that are acceptable. An organization can use its strong culture to enhance diversity if it seeks out and hires diverse individuals because of their alternative strengths. If they effectively encourage the unique strengths that diverse people

bring to the organization, they eliminate any institutional bias or insensitivity to people who are different. 7. What benefits can socialization provide for the organization? For the new employee? Answer Most of the benefits are mutual rather than distinct. New employees are not fully indoctrinated in the organizations culture. They are unfamiliar with the organizations culture and are potentially likely to disturb the beliefs and customs that are in place. Socialization is the organization helping new employees adapt to its culture. Metamorphosis and the entry socialization process is complete when the new member has become comfortable with the organization and his job. Exhibit 18-2 shows how successful metamorphosis should have a positive impact on the new employees productivity and his commitment to the organization, and reduce his propensity to leave the organization. 8. How is language related to organizational culture? Answer Many organizations and units within organizations use language as a way to identify members of a culture or subculture. By learning this language, members attest to their acceptance of the culture and, in so doing, help to preserve it. Organizations, over time, often develop unique terms to describe equipment, offices, key personnel, suppliers, customers, or products that relate to their business. New employees are frequently overwhelmed with acronyms and jargon that, after six months on the job, have become fully part of their language. Once assimilated, this terminology acts as a common denominator that unites members of a given culture or subculture. 9. How can management create an ethical culture? Answer An organizational culture most likely to shape high ethical standards is one that is high in risk tolerance, low to moderate in aggressiveness, and focuses on means as well as outcomes. If the culture is strong and supports high ethical standards, it should have a very powerful and positive influence on employee behavior. An organization can create an ethical culture by doing the following: Be a visible role model. Employees will look to top-management behavior as a benchmark for defining appropriate behavior. Communicate ethical expectations. Ethical ambiguities can be minimized by creating and disseminating an organizational code of ethics. Provide ethical training. Use training sessions to reinforce the organizations standards of conduct, to clarify what practices are and are not permissible, and to address possible ethical dilemmas. Visibly reward ethical acts and punish unethical ones. Performance appraisals of managers should include a point-by-point evaluation of how his or her decisions measured against the organizations code of ethic s. Provide protective mechanisms. The organization needs to provide formal mechanisms so that employees can discuss ethical dilemmas and report unethical behavior without fear of reprimand. This might include creation of ethical counselors, ombudsmen, or ethical officers. 10. What criticisms have been targeted against bringing spirituality to the workplace? Answer Critics of the spirituality movement in organizations have focused on two issues: 1) Do organizations have the right to impose spiritual values on their employees? and 2) Are spirituality and profits compatible? This criticism is undoubtedly valid when spirituality is defined as bringing religion and God into the workplace. However, the goal is limited to helping employees find meaning in their work lives and to

use the workplace as a source of community. The issue of whether spirituality and profits are compatible objectives is certainly relevant for managers and investors in business. A recent research study by a major consulting firm found that companies that introduced spiritually based techniques improved productivity and significantly reduced turnover. QUESTIONS FOR CRITICAL THINKING 1. Is socialization brainwashing? Explain. Answer Yes and no. Yes, in that it is helping or coercing the individual into adjusting his/her behavior to match the expectations of the organization. On the other hand, socialization is necessary in order to work together. Imagine a kindergarten class if the students were not socialized. No matter how good a job the organization does in recruiting and selection, new employees are not fully indoctrinated in the organizations culture. Most importantly, because they are unfamiliar with the organizations culture, new employees are potentially likely to disturb the b eliefs and customs that are in place. The organization will, therefore, want to help new employees adapt to its culture. 2. If management sought a culture characterized as innovative and autonomous, what might its socialization program look like? Answer Southwests. They would encourage risk -taking, provide a maximum of freedom to managers and employees, and tie rewards, raises, and promotions to innovative behavior, initiative, and innovation. 3. Can you identify a set of characteristics that describes your colleges culture? Compare your assessment with those of several of your peers. How closely do they agree? Answer The answer will vary by school. The key is conducting an open discussion and not reacting to students assessments, yet making them defend them with some objective evidence, not just subjective experience. 4. Todays workforce is increasingly made up of part-time or contingent employees. Is organizational culture really important if the workforce is mostly comprised of temporaries? Answer It can be argued that organizational culture is particularly important with a temporary or contingent workforce because it helps to convey a sense of identity for organization members. It is the social glue that helps hold the organization together and it enhances social system stability. The shared meaning of a strong culture ensures that everyone is pointed in the same direction. 5. We should be opposed to the manipulation of individuals for organizational purposes, but a degree of social uniformity enables organizations to work better. Do you agree or disagree with this statement? What are its implications for organizational culture? Discuss. Answer Students responses will vary.

1. What is meant by the phrase we live in an age of discontinuity?


Answer It means that we live in an age of change and disconnection from the past. Things are changing quickly and in ways we could not anticipate, and that these changes have wide-ranging effects. Such as, beginning in the early 1970s, with the overnight quadrupling of world oil prices, economic shocks have continued to impose changes on organizations. Or, the recent economic problems in Russia, Asia, and Latin

America rocked world stock markets.

2. Resistance to change is an irrational response. Do you agree or disagree? Explain.


Answer One of the most well-documented findingsorganizations and their members resist change. It is rational. It is just that resistance to change doesnt seem rational because it doesnt necessarily surface in standardized ways. Resistance can be overt, implicit, immediate, or deferred. Implicit resistance efforts are more subtleloss of loyalty to the organization, loss of motivation to work, increased errors or mistakes, increased absenteeism due to sicknessand hence more difficult to recognize. A change may produce what appears to be only a minimal reaction at the time it is initiated, but then resistance surfaces weeks, months, or even years later. A single change that might have little impact becomes the straw that breaks the camels back. Reactions to change can build up and then explode in some response that seems totally out of proportion to the change action it follows. The resistance, of course, has merely been deferred and stockpiled. What surfaces is a response to an accumulation of previous changes.

3. Why is participation considered such an effective technique for lessening resistance to change?
Answer Its difficult for individuals to resist a change decision in which they participated. Pri or to making a change, those opposed can be brought into the decision process. Assuming that the participants have the expertise to make a meaningful contribution, their involvement can reduce resistance, obtain commitment, and increase the quality of the change decision.

4. Why does change so frequently become a political issue in organizations?


Answer Because change invariably threatens the status quo, it inherently implies political activity. Internal change agents typically are individuals high in the organization who have a lot to lose from change. What if they are no longer the ones the organization values? Politics suggests that the impetus for change is more likely to come from outside change agents. Managers who have spent their entire careers with a single organization and eventually achieve a senior position in the hierarchy are often major impediments to change. You should expect that long-time career executives will be sources of resistance. This, incidentally, explains why boards of directors that recognize the imperative for the rapid introduction of second-order change in their organizations frequently turn to outside candidates for new leadership.

5. How does Lewins three-step model of change deal with resistance to change?
Answer To deal with that resistance, management could use positive incentives to encourage employees to accept the change. Management might also consider unfreezing acceptance of the status quo by removing restraining forces. Employees could be counseled individually. Each employees concerns and apprehensions could be heard and specifically clarified. Assuming that most of the fears are unjustified, the counselor could assure the employees that there was nothing to fear and then demonstrate, through tangible evidence, that restraining forces are unwarranted. If resistance is extremely high, management may have to resort to both

reducing resistance and increasing the attractiveness of the alternative if the unfreezing is to be successful.

6. What changes can an organization that has a history of following the leader make to foster
innovation? Answer Change refers to making things different. Innovation is a more specialized kind of change. Innovation is a new idea applied to initiating or improving a product, process, or service. There are several sources of innovation. Structural variables include developing an organic structure, providing sufficient resources to permit innovation, and increasing communication. Managers should: encourage experimentation. reward both successes and failures. celebrate mistakes. actively promote the training and development. offer high job security so employees dont fear getting fired for making mistakes. encourage individuals to become champions of change.

7. Learning organizations attack fragmentation, competitiveness, and reactiveness. Explain this


statement. Answer A learning organization is an organization that has developed the continuous capacity to adapt and change. Proponents envision it as a remedy for the three fundamental problems inherent in traditional organizations: fragmentation, competition, and reactiveness. First, fragmentation based on specialization creates walls and chimneys that separate different functions into independent and often warring fiefdoms. It fosters communication. Second, an overemphasis on competition often undermines collaboration. It emphasizes cooperation and coordination. Third, reactiveness misdirects managements attention to problem solving rather than crea tion. It has a proactive perspective.

8. How does an organization build a knowledge-management system?


Answer An organization builds a knowledge management system as it records the knowledge and expertise of its employees and makes that information easily accessible. It does this by developing computer databases of pertinent information that employees can readily access, by creating a culture that supports and rewards sharing, and developing mechanisms that allow employees who have developed valuable expertise and insights to share them with others.

9. How are opportunities, constraints, and demands related to stress? Give an example of each.
Answer Students examples will vary but should take into consideration the following facts. Stress is a dynamic condition in which an individual is confronted with an opportunity, constraint, or demand related to what he/she desires and for which the outcome is perceived to be both uncertain and important. Opportunities permit individuals to use stress positively to rise to the occasion and perform at or near their maximum. Typically, stress is associated with constraints and demands. The former prevent you from doing what you desire. The latter refers to the loss of something desired. For opportunities to cause stress, there must be uncertainty over the outcome and the outcome must be important.

10. What can organizations do to reduce employee stress?


Answer Strategies that management might want to consider include improved personnel selection and job placement, use of realistic goal setting, redesigning of jobs, increased employee involvement, improved organizational communication, and establishment of corporate wellness programs. QUESTIONS FOR CRITICAL THINKING 1. How have changes in the workforce during the past 20 years affected organizational policies? Answer A multicultural environment requires greater sensitivity to individual differences requiring training in diversity. Human resource policies and practices changed to attract and keep this more diverse workforce. Large expenditure on training to upgrade reading, math, computer, and other skills of employees.

2. Managing today is easier than at the turn of the century because the years of real change took place
between the Civil War and World War I. Do you agree or disagree? Discuss. Answer Begin by having students brainstorm changes they know of that took place between 1860 and 1918. Then brainstorm changes from 1918 until today. List these items on the blackboard. Be prepared to prompt students, because many will not have a timeframe for the changes they note; i.e., electricity, telegraph, the railroad, the car, the airplane, etc.

3. Are all managers change agents? Discuss.


Answer They can be to the degree they change one or more of the following elements in response to changing conditions in the business environment. 1) Changing conditions demand structural changes. Change agents can alter one or more of the key elements in an organizations design. 2) Competitive factors or innovations within an industry often require change agents to introduce new equipment, tools, or operating methods. 3) The layout of work space is the result of thoughtful consideration of work demands, formal interaction requirements, and social needs. 4) Changing people typically involves changing the attitudes and behaviors of organizational members through processes of communication, decision making, and problem solving.

4. Discuss the link between learning theories discussed in chapter 2 and the issue of organizational
change. Answer Learning is any relatively permanent change in behavior that occurs as a result of experience. We infer that learning has taken place if an individual behaves, reacts, responds as a result of experience in a manner different from the way he formerly behaved. The text definition has several components that deserve clarification. First, learning involves change. Second, the change must be relatively permanent. Third, our definition is concerned with behavior. Finally, some form of experience is necessary for learning. Learning theory closely parallels Lewins three-step change process. Unfreezing the status quo Movement to a new state Refreezing the new change to make it permanent See Exhibit 19-6.

5. Do you think napping on the job is an acceptable practice in the workplace? What negatives do you
see, if any, in promoting this practice? Answer . Students answers will vary on thisbut most employers have policies against it. Why do they think that is the case? Is napping a stress reliever or theft (assuming the employee is being paid while sleeping)?

You might also like